Как изучать математику, чтобы действительно понимать ее и вести здоровый образ жизни при наличии свободного времени? [закрыто]

Вот моя проблема, с которой я столкнулся; Я очень много работал над изучением математики, поэтому я начал понимать, что лучше понимаю вещи. Однако за это приходится платить: за последние несколько лет у меня практически не было физических упражнений, я набрал 30  кг, я провел бесчисленное количество часов за учебой по ночам, постоянно недосыпал, потерял социальную жизнь и у меня появились проблемы со здоровьем. У меня хорошие оценки, но я чувствую, что зря трачу свою жизнь.

Я люблю математику, когда она делается по-моему, но это редко бывает. Я бы очень хотел, чтобы моя карьера была сосредоточена вокруг математики (топологии, алгебры или чего-то подобного). Я хочу действительно понимать вещи, и я хочу, чтобы доказательства были сделаны (разумно) строго. Раньше меня обвиняли в формалисте, но я себя таковым не считаю. Однако я признаю, что я перфекционист. Для сравнения, абсолютно строгими для меня являются ответы Тео, Артуро, Джима Белка, Мариано и др. Из моего опыта, 80 % или больше математики в нашей школе делается отрывочно, типа «Хм, наверное, правда» (точно так же, как чтение кулинарных рецептов), что чертовски раздражает меня. Большинство одноклассников приспосабливаются к этому, но я почему-то не могу. Я не понимаю вещей, если не понимаю их (почти) полностью. Они узнали, «как надо поступать», но реже задаются вопросом, ПОЧЕМУ это правильно. У меня есть два знакомых физика, у которых точно такая же проблема. Один на уровне докторантуры, постоянно срывающийся, а другой вообще забросил физику после получения диплома. Кроме одного 8 , у него был идеальный послужной список, все 10 с. Он упомянул, что, по его мнению, недостаточно хорошо понимает физику. По моему опыту, ВСЕ его одноклассники понимают меньше, чем он, они просто плывут по течению и принимают определенные утверждения за истину. Удалось ли вам изучить все вовремя, И достаточно тщательно, чтобы вы могли это понять? **

ДОПОЛНЕНИЯ:

Часто я единственный, кто находит серьезные проблемы в доказательствах, формулировках теорем и отработанных упражнениях на занятиях. Либо все остальные все понимают, большинство, либо не понимают и не заботятся о возможных проблемах. Часто я нахожу дыры в доказательствах и отсутствие гипотез в теоремах. Когда я предъявляю их профессору, он говорит, что я прав, и упоминает, что я очень точен. Как это точно, если теорема не выполняется в ее нынешнем состоянии? Мы вообще должны понимать доказательства? Являются ли доказательства действительно просто набросками? Как же тогда человек может открывать математические истины? Является ли изучение математики всего лишь одной большой шуткой, и вы не должны воспринимать ее слишком серьезно?

ПРИМЕЧАНИЕ:

У меня есть куча видов спорта, которые я люблю и которыми раньше занимался. Более того, раньше у меня была отличная социальная жизнь, так что вам не нужно давать советы по этому поводу. Я не общаюсь и не занимаюсь спортом, потому что переваривание доказательств и попытки понять идеи, стоящие за всем этим, съедают все мое время. Если я пойду в поход, это заберет 2 дней, один на прогулку + один на отдых и восстановление. Если я пойду тренироваться в ММА, я не буду сосредоточен на весь день. Я не могу мгновенно переключиться с бокса на погоню за диаграммами. Кроме того, я не могу просто заниматься полчаса. Я учусь следующим образом: я открываю книгу, ищу то, что уже знаю, но забыл из предыдущего дня, а затем иду от теоремы к теореме, от доказательства к доказательству, исправляю ошибки, добавляю пояснения и т. д. и т. д. , У меня есть плохая привычка с трудом начинать что-то. Однако, когда я завожу двигатель, я запускаю «мой двигатель» и с трудом останавливаюсь., особенно если все идет хорошо. Вот почему я непреднамеренно трачу час или два перед учебой на самые не относящиеся к делу вещи, просто чтобы не учиться. Это происходит, особенно когда у меня было больше математики, чем я могу засунуть себе в глотку, а это у меня есть, чтобы мысленно подготовиться к учебе. Но, так как мой двигатель действительно заводится и учеба идет хорошо (многое доказал, многое понял), мне трудно остановиться, поэтому я часто остаюсь поздно ночью, до 4 утра, 5 утра и 6 утра. Когда день приходит экзамен, я вообще не сплю, и день и ночь меняются местами . Я ложусь спать в 13 часов и просыпаюсь в 21 час... Я знаю, что это нехорошо, но я не могу избавиться от этой привычки. Если я бесполезен весь день,. Я знаю, что этого не должно происходить, если кто-то любит математику. Однако, когда тебе «навязывают», что, сколько и за какое время ты должен изучать, тебя начинает отталкивать математика. Математика перестает быть развлечением/развлечением и становится тяжелой работой, которую просто необходимо делать.введите описание изображения здесь

Уместно замечание Джан-Карло Рота: «Факты математики проверяются и представляются аксиоматическим методом. Однако следует остерегаться путать представление математики с содержанием математики . Аксиоматическое представление математического факта отличается от от того, что преподносится как лекарство, отличающееся от еды. Это правда, что именно это лекарство необходимо, чтобы держать математиков на безопасном расстоянии от самообманов ума. Тем не менее понимать математику означает уметь забыть лекарство и наслаждайтесь едой».
Вышеизложенное взято из стр. 96 « Недискретной математики» Рота .
@Bill: нескромные мысли.
Что касается «ведения здорового образа жизни», я обнаружил, что закон Паркинсона определенно применим. Точно так же, как я нахожу необходимость «назначить встречу с моим исследованием» (выделение определенного количества времени в большинстве дней, чтобы подумать о моем исследовании, независимо от того, делаю ли я успехи или нет), я также обнаруживаю, что если я откладываю время для физических упражнений / питания / и т. д., я могу использовать его для этого, но если я не отложу его в сторону, это не значит, что у меня есть куча времени, чтобы я ничего не делал. Отложите время.
Я бы попытался получить обзор того, что я изучаю в первую очередь. Я хотел бы думать об этом как о большом холсте; заполните детали в соответствии с тем, что вас интересует. Вы абсолютно точно не можете научиться всему (даже близко), но вы можете учиться сверху вниз, а не снизу вверх. Для ваших комментариев о строгости вам может быть интересно: cheng.staff.shef.ac.uk/morality/morality.pdf
Обратите внимание, что недостаток сна может затруднить запоминание вещей. Попробуйте больше спать (т. е. не менее 8 часов в сутки), это сделает вас более продуктивным и может даже дать вам больше времени, чем вы тратите на сон, потому что не так много забываете.
Это действительно задело здешнее сообщество; никакие другие вопросы не дают 6 длинных ответов за 6 часов! Так что @Leon: Цените тот факт, что вы не одиноки в этой ситуации, пока не найдете лекарство. :)
Скорее всего, источником ваших проблем является недостаток сна. Я считаю, что невозможно быть эффективным без хорошего отдыха. (То есть, мне нужны мои 9 часов) Кроме того, думайте о математических задачах прямо перед сном, так они могут присниться вам, и часть вашего ума все еще будет работать. Буквально на прошлой неделе, лежа в постели, пытаясь заснуть, мне пришла в голову идея решить проблему, о которой я думал около 6 часов ранее в тот день.
Я нашел эту статью «Руководство по выживанию математика» Питера Г. Казаццы, «предназначенную как руководство по выживанию для тех студентов, учителей и математиков, у которых возникают проблемы с интерпретацией математического опыта». Может быть, это может помочь вам.
Я считаю, что прогулка помогает мне думать и соображать вещи прямо в голове. Ходьба помогает мне решать проблемы. Кроме того, это не совсем марафон, но много прогулок дают мне достаточно упражнений!
Что касается вашей памяти, то анекдот об Эрдосе может облегчить ваш ум: он изо дня в день забывал определения и спрашивал: «Что такое Хаусдорф…», а затем ему это объясняли, и он доказывал теорему под рукой. , на следующий день кто-то говорит: "Пусть Икс быть пространством Хаусдорфа», и Эрдос спрашивает: «Что такое Хаусдорф…». Это не просто запоминание всего, а скорее усвоение техник и понимание общей картины. (И это всегда помогает быть Эрдосом :P)
Я не помню, чтобы писал это, но я должен был, потому что вы описываете меня. Я страдал от стресса на протяжении всего колледжа, некоторые способы, которыми я обходился, заключались в том, чтобы ложиться спать к полуночи, независимо от того, сколько у меня было домашних заданий (иначе я бесполезен) и 15-минутной прогулки, даже если это было только к моей машине. Я начал практиковать медитацию по 8 минут в день, и это, похоже, помогло. И я работал в библиотеке, чтобы делать домашнюю работу на работе. То, что я учился в школе, действительно навредило моим отношениям с моей девушкой, но мы помирились и потом поженились. Мой самый главный совет — не переживайте из-за оценок, ха-ха, нет?
"Как изучать математику... и иметь... свободное время?" Решение: занимайтесь математикой в ​​свободное время.
Занимайтесь математикой в ​​свободное время и ориентируйтесь на расслабление, ничего страшного, если вы что-то пропустите. Простая тяжелая работа не будет работать в математике
Треугольник довольно красивый и правдивый.
У меня есть идея, но я не уверен, что она поможет. Я даже не знаю, не усугубит ли это ситуацию, так что оставлю на ваше усмотрение судить, думаете ли вы, что от этого станет хуже. Одна из идей — прочитать ответы на math.stackexchange.com/questions/1333206/… . Другая идея — придумать собственную систему чистой теории чисел. Я думаю, что каждый способен продолжать получать новые фрагменты информации и сохранять их с ненулевой скоростью, даже если это очень медленный темп. Чтобы тренировать свою память, вы могли бы продолжать выяснять больше утверждений, доказуемых в этой системе.
в вашей голове. Я думаю, что время от времени вы поймете одно утверждение, и вы поймете точно такое же утверждение и забудете его так много раз до этого, что вы, наконец, сохраните его на этот раз. Не записывая то, что вы выяснили, вы можете постепенно лучше запоминать новые утверждения, которые вы выясняете, и выяснять новые утверждения, которые вы будете запоминать немного чаще, чем раньше. Вы можете подумать, что я не могу понять никаких утверждений, которые действительно полезны, и чувствуете себя застрявшим. Это не имеет значения. Все же лучше придумать еще одно утверждение и сохранить его, чем нет.
более. Может быть, поскольку утверждения, которые вы ранее считали бесполезными, могут быть использованы для понимания других утверждений, вы в конечном итоге выясните достаточно утверждений, чтобы понять так много, а затем поймете, что эти утверждения в конце концов не были бесполезными, потому что они в конечном итоге приводят к полезным открытиям. . Если система достаточно проста, и вместо этого вы получаете эффект, противоположный тому, что вы утверждаете, что получили полное понимание системы, и она кажется такой простой, то вы можете расширить систему до более сильной системы, чтобы описать утверждение, которое доказывается всеми утверждениями, которые эта система доказывает. верны.
Наконец, моя третья идея основана на моей неуверенной гипотезе о том, что Math Overflow подходит для вопросов, которые побуждают к полезным математическим исследованиям, которые не слишком сложны для выполнения, и состоит в том, чтобы переместить этот вопрос в MathOverflow, чтобы стимулировать исследования о том, как создать курс, подходящий для таких людей, как вы. . Если они не примут этот вопрос, то, возможно, вы могли бы сначала задать вопрос на Academia Stack Exchange, чтобы узнать больше о том, почему они учат так, как они это делают, или почему мозг людей учится определенным образом, тогда, возможно, у вас будут ресурсы, чтобы задать вопрос. этот вопрос на MathOverflow. Если вы исследователь, возможно, вы также можете
спросите об этом на ResearchGate. Если нет, возможно, вы могли бы задать вопрос на Stack Exchange, чтобы получить информацию, которая поможет вам понять, как вы могли бы стать исследователем, но я не знаю, какой веб-сайт Stack Exchange.
Я также думаю, что, вероятно, лучше не получать никакой помощи от компьютерной программы, подобной Python, для выполнения расчетов для вас или поиска Google, чтобы найти ответы на вопросы. Возможно, эти ответы — ответы, которые вам на самом деле не нужны, потому что вы всегда сможете мысленно придумать еще одно утверждение и запомнить его, но их использование приучит вас не придумывать никаких утверждений самостоятельно. Python не может сделать за вас творческое мышление и дать вам истинное математическое утверждение, эквивалентное письменному ответу на английском языке, которое научит вас, как действительно хорошо разбираться в математике. Может быть
доказуемое математическое утверждение о том, что как только вы его изучите, вы сможете понять очень искусственный интеллект, конкретную математическую технику, но у вас, вероятно, больше шансов найти ее, если вы не получите помощь от Python, чем если бы вы это сделали. Согласно matheducators.stackexchange.com/questions/7718/… , люди, изучающие сложные книги в молодом возрасте, вероятно, с меньшей вероятностью смогут разобраться во всем самостоятельно и застрять, когда у них возникнет вопрос, который они не могут решить. найти книгу или результаты поиска Google, которые отвечают.

Ответы (27)

На мой взгляд, центральный вопрос, который вы должны задать себе, заключается в том, какова конечная цель вашего обучения. Например, американская студенческая жизнь, изображенная в фильме, гедонистична и, конечно же, не сосредоточена на реальной учебе. Ваш пример — полная противоположность — вы описываете себя как подвижника, преданного учености.

Многие люди считают важным вести сбалансированный образ жизни. Если такой человек столкнулся с вашей ситуацией, он мог бы искать какой-то компромисс, например, тратить меньше времени на учебу в обмен на более низкие оценки. Если что-то пойдет не так, они могут подумать об отказе от всего предприятия. Ваша точка зрения может быть иной – для вас самое важное измерение – это интеллектуальный рост, ради которого вы готовы пожертвовать всем.

В другом ответе упоминалось, что здоровый образ жизни может способствовать вашей учебе. Люди склонны «выгорать», если работают слишком усердно. Я знал таких людей, и им приходилось периодически «прохлаждаться» где-нибудь далеко. Наоборот, неучебная деятельность может быть бодрящей и освежающей.

Другой похожий аспект - это «занятость». Некоторые люди обнаруживают, что благодаря многозадачности они становятся более продуктивными в каждом из своих индивидуальных «фронтов». Но такой стиль жизни не для всех.

Возвращаясь к моему исходному вопросу, чего вы ожидаете добиться, успешно учась в школе? Вы нацелены на академическую карьеру? Профессиональная карьера? В Северной Америке высшее образование стало обрядом посвящения, который многие выпускники находят очень проблематичным из-за связанных с этим затрат. Для них вопрос зачастую экономичен – образование в Северной Америке дорогое.

Возможно, вы обнаружите, что, закончив учебу, вы должны повернуть свою жизнь в какое-то совсем другое русло. Вы можете осознать, что потратили впустую несколько лучших лет своей жизни, усердно учась, исключая все остальное, усилия, которые в конечном итоге ни к чему вас не приведут. Это наихудший сценарий.

Более конкретно, я предлагаю вам планировать заранее и подумать, стоит ли оно того. Это требует как серьезной оценки собственной ценности, так и некоторых предположений о будущем рынке труда. Вы также должны оценить, насколько важными вы собираетесь считать эти нынешние исследования в своем будущем - как с экономической, так и с «культурной» точки зрения.

Все это может звучать обескураживающе, но ваша ситуация, как вы ее описываете, весьма плачевна. Мало того, что вас это не устраивает, так еще и для стороннего наблюдателя это выглядит проблематично. Однако я подозреваю, что вы преувеличиваете, рассматривая ситуацию с романтической, героической точки зрения. Поэтому лучше всего поговорить с людьми, которые знают вас лично.

Еще лучше поговорите с людьми, которые старше вас и находятся на следующем этапе «жизни». У них более широкий взгляд на вашу ситуацию, которую они из своих знакомых просто еще живо помнят. Однако даже к их рекомендациям следует относиться с долей скептицизма, поскольку их нынешние заботы — лишь часть более широкой картины, всеобъемлющей «жизни».


Наконец, несколько слов, более относящихся к обсуждаемой теме.

Во-первых, стратегия обучения. Я думаю, что лучший способ научиться — это решать сложные задачи. Совет, данный здесь, пытаясь «реконструировать» учебник перед его чтением, кажется очень трудоемким и, на мой взгляд, концентрирует усилия не на том месте.

То же самое касается запоминания теорем — иногда действительно «понять» доказательство теоремы можно, только изучив более сложную тему. Даже исследователь, который первоначально выступил с доказательством, вероятно, не понимал его «на самом деле», пока не была разработана более широкая перспектива.

Запоминание теорем — это не ваш выбор, а скорее необходимость. Мне всегда не нравилось срыгивание, и очень жаль, что вам его навязывают. Я рад, что вместо этого моя школа давала нам настоящие проблемы для решения — в любом случае это гораздо ближе к исследованиям. Поскольку вам предстоит пройти через этот прискорбный процесс, попробуйте придумать метод запоминания, который имеет и другие преимущества — возможно, он направлен на лучшее понимание того, «что происходит», а не на сами шаги. Это важный навык.

Во-вторых, один из ответов предлагает попытаться вывести как можно больше теорем в качестве «математической» вещи, которую следует сделать, увидев определение. Я бы предложил скорее наоборот: сначала выясните, что влечет за собой определение, а затем попытайтесь понять, почему это понятие было определено в первую очередь и почему именно таким образом.

В математике принято начинать изучение предмета с длинного списка «важных определений», которые на данном этапе не имеют никакого значения. Вы поймете предмет, когда сможете объяснить, откуда берутся эти определения, какие объекты они описывают; и когда вы можете «чувствовать» эти объекты интуитивно. Это далеко от возможности вывести некоторые факты, которые более или менее прямо следуют из определений.

@Yuval: Очень хорошо написано (даже пришлось искать некоторые слова), и, прежде всего, отличный совет, я очень благодарен за ваши предложения и ваше время. Для меня вопрос конечно не экономический, так как учеба здесь бесплатная. «Мне всегда не нравилось срыгивание, и очень жаль, что вам это навязывают». Нет нет, нас конечно никак не заставляют запоминать вещи как в обществознании, но с каждым новым предметом предполагается, что мы знаем большую часть материала из предыдущих, а это очень много, а я постоянно освежаю и забываю .
Это действительно расстраивает, что я понимаю тему, и через 6 месяцев она почти вылетела из моей головы. Тем более, что память у некоторых одноклассников куда лучше. Конечно, когда я переучиваю, все идет более гладко и быстро, но дело в том, что мне приходится переучиваться снова и снова. А поскольку количество материала только увеличивается, это становится все труднее и труднее. Есть много-много областей математики, и количество всех теорем, лемм, утверждений и определений огромно. Конечно, интуиция всегда помогает, но чудес она не творит. Как и в спорте, без кардио спортсмен обречен.
@Yuval: Кстати, я почти не заметил, что был еще один (ваш) ответ. Мне сказали, что использование @ информирует человека о другом комментарии, адресованном ему. Где/как я их замечаю, не просматривая каждый раз все свои вопросы?
@Leon: Проведение математических исследований предполагает накопление огромного количества знаний и постоянное их использование; не обязательно путем обращения к неясной лемме или даже умеренно важной теореме, а скорее через аналогии и интуитивное понимание природы некоторых математических объектов и методов их рассуждения. Если вы действительно обнаружите, что продолжаете забывать старый материал — и под этим я подразумеваю основы, а не продвинутые результаты — вы, вероятно, учитесь либо слишком быстро, либо слишком много, либо слишком нервно.
Что касается вашего технического вопроса, я полагаю, что вы получаете уведомление - появляется какой-то специальный сигнал вверху страницы. И если вы отметите правильное поле при задании вопроса, вы получите уведомление по электронной почте, когда кто-либо ответит на ваш вопрос.
Я думаю, что в основном не забываю основы, но чтобы применить определенную лемму, нужно знать гипотезы. В топологии, алгебре и анализе много подводных камней, много контрпримеров, наглядно демонстрирующих, как легко сделать неверный вывод.
Требуется время, чтобы привыкнуть к новой форме знаний, особенно такой сложной, как математика. В какой-то момент вы выйдете на новый уровень понимания — вдруг весь предыдущий материал, который раньше казался таким сложным, станет откровенно простым. Вы начнете запоминать концепции и основные результаты вскоре после решения соответствующих упражнений. Это может показаться недоверчивым, но если вы будете настойчивы, это в конце концов произойдет. Будьте терпеливы и расслаблены. Все когда-то были в вашей нынешней ситуации. В конце концов все становится лучше.
Возможно, это не самый высокий ответ (пока), но, на мой взгляд, он, безусловно, самый всесторонний. Я согласился с наблюдением, что некоторые из других методов, описанных на этой странице, требуют много времени. Я пробовал их, и в итоге вы выучите гораздо меньше, чем если бы вы, скажем, просто изучили какой-то базовый материал и попробовали несколько упражнений, постепенно увеличивая сложность каждый раз.
Чем занимался @YuvalFilmus между 2002 (окончание магистратуры) и 2009 (начало докторантуры)?

Позвольте мне сказать вам, что единственное, чем я занимаюсь последние четыре года своей жизни, — это математика. Я полностью наслаждался этим опытом, но у меня также были моменты, когда я был несколько неуверен относительно того, как подойти к моему обучению. Я думаю, что нет единого правила, которое работает для всех; тем не менее, позвольте мне ответить на некоторые ваши вопросы. Я надеюсь, что смогу помочь:

Вопрос: Как правильно изучать математику?

Ответ : Я думаю, что лучший способ изучения математики заключается в следующем. Предположим, вы уже выбрали книгу по математике по теме, которую вам действительно интересно изучать. Когда вы читаете книгу, старайтесь активно думать о предмете с разных точек зрения . Например, если представлено определение, потратьте не менее 30 минут на обдумывание определения. Если вы изучаете книгу по линейной алгебре и в ней дается определение «нильпотентного оператора», вам следует попытаться самостоятельно открыть для себя некоторые основные свойства нильпотентных операторов, не читая дальше. Поначалу это может быть сложно, но, в конечном итоге, в исследовательской математике важна способность делать это эффективно с максимально возможным количеством определений.

Возьмем следующий пример из элементарной теории групп. Автор дает определение максимальной подгруппы конечной группы г : подгруппа М из г называется максимальной подгруппой, если М является собственной подгруппой г и если нет собственных подгрупп г строго содержащий М . Вы должны попытаться предпринять следующие шаги:

(1) Найдите примеры максимальных подгрупп в конечных группах и начните с самых простых примеров ! Например, тривиальная группа не может иметь максимальной подгруппы. Если вы понимаете это, вы поняли один пункт определения. Следующим шагом является рассмотрение простейших циклических групп. Каковы максимальные подгруппы циклической группы второго порядка? Каковы максимальные подгруппы циклической группы 4-го порядка? Подумайте об основных примерах, таких как этот. Когда будете готовы, попробуйте самостоятельно сформулировать общую теорему о максимальных подгруппах циклической группы порядка н . Вы должны прийти к теореме о том, что подгруппа ЧАС циклической группы г максимально тогда и только тогда, когда число | г | | ЧАС | является простым.

Продолжайте искать другие примеры максимальных подгрупп в конечной группе. Следующим шагом является рассмотрение 4-группы Клейна и групп перестановок низких порядков. Я надеюсь, что к этому моменту вы действительно увлеклись концепцией максимальной подгруппы. Сначала это определение может показаться чем-то произвольным; однако теперь, когда вы подумали об этом, вы начали обретать чувство «собственности» над определением .

(2) Теперь пришло время сформулировать и доказать некоторые теоремы о максимальных подгруппах . Опять же, подумайте о самых простых примерах. Одна вещь, которая может обескуражить новичка, — это неспособность ответить на вопрос, который кажется простым в течение длительного периода времени. Какой хороший пример простой теоремы? Вы можете изучать те конечные группы, которые имеют ровно одну максимальную подгруппу. Что вы можете сказать о такой группе? Если вы обнаружите, что застряли, попробуйте вернуться к примерам максимальных подгрупп, которые вы придумали ранее. На самом деле на этот вопрос можно ответить вполне удовлетворительно; конечная группа с единственной максимальной подгруппой является циклической простого степенного порядка.

(3) Следующим шагом является предположение о некоторых других свойствах максимальных подгрупп на основе примеров, которые вы придумали в (1) . Например, вы выяснили, что если ЧАС является максимальной подгруппой конечной циклической группы г , затем | г | | ЧАС | является простым числом. Это верно для всех групп? г ? Вы можете думать о группах г для чего это верно?

Заметили, как можно деконструировать простое определение, чтобы прийти к множеству интересных вопросов ? Это то, чем математик занимается постоянно, и это очень важный навык. Поначалу это может показаться трудным, но это сделает математику еще более увлекательной и даст вам чувство «собственности» над содержанием. Вы разработали эту часть математики . Именно так я изучаю математику и могу с уверенностью сказать вам, что если вы будете практиковать это, то скоро это станет нормой.

Что вы делаете после того, как посмотрите на определение и тщательно обдумаете его? Вы продолжаете читать текст. Есть большая вероятность, что вы заметите, что автор констатирует некоторые из результатов, которые вы обнаружили самостоятельно. Если повезет, будут результаты, о которых автор не заявил. Если это так, было бы неплохо спросить (например, на этом сайте) об оригинальности результата.

Однако вы столкнетесь с теоремами, касающимися определений, о которых вы просто не подумали. Вы должны сопротивляться искушению увидеть доказательства этих теорем и лучше попытаться доказать эти теоремы самостоятельно . Подумайте над теоремой хотя бы несколько часов, прежде чем сдаться. Обратите внимание, что теоремы с довольно короткими доказательствами могут потребовать очень оригинальных идей, и поэтому вам не следует заставлять себя доказывать теорему за небольшой промежуток времени.

Поначалу вам придется долго доказывать некоторые теоремы. Будут рутинные теоремы, и они должны быть доказаны довольно быстро. Но будут и трудные теоремы. По мере накопления опыта ваше мышление будет ускоряться, и эти теоремы будут даваться вам легче . Тем не менее, вы не должны ожидать, что это будет иметь место изначально.

Например, вы можете столкнуться со следующей теоремой линейной алгебры: если Н является нильпотентным линейным преобразованием из векторного пространства В самому себе, и если размерность В является н , затем Н н "=" 0 . Самостоятельное доказательство этой теоремы — очень ценный и полезный опыт . Если вы еще этого не видели, предлагаю вам попробовать это доказать. Однако это не так уж сложно.

Вопрос: Как не забыть математику?

Ответ : Я тоже забывал математику, когда изучал ее. Я говорил об этом с разными математиками, и они говорили то же самое. Дело в том, что вы просто должны принять с самого начала, что вы забудете то, чему научились. Однако есть способы свести это к минимуму.

Например, лучший способ не слишком беспокоиться о том, что вы забудете математику, — это решить ее самостоятельно . Например, рассмотрим шаги, которые я предложил в предыдущем вопросе. Даже если вы это сделаете, вы все равно можете забыть о математике, особенно если рассматриваемый результат было довольно легко доказать. (Обратите внимание, однако, что если результат трудно доказать, и вы потратите, допустим, 10 часов, чтобы доказать его, то вы, вероятно, никогда не забудете его до конца своей жизни .)

Лучший способ — записать всю математику, которую вы изучаете . Делайте подробные заметки. Например, когда в прошлом году я прочитал «Реальный и комплексный анализ» Уолтера Рудина, я сделал целых 3 книги заметок. На самом деле, я записал 600 страниц математики, когда прочитал только 315 страниц!

Запишите каждое определение, каждую теорему и каждое доказательство . Определения и теоремы должны быть воспроизведены дословно из книги, поскольку важно убедиться, что вы правильно понимаете строгость. Однако доказательства следует писать своими словами.

Вопрос: Как вести здоровый образ жизни?

Ответ : Боюсь, у меня действительно нет хорошего ответа на этот вопрос. За четыре года, что я изучаю математику, я точно ничем другим не занимался. Поэтому я не могу дать совет, как распоряжаться своим временем. Если вы серьезно изучаете математику, вы обнаружите, что тратите практически весь день на изучение предмета. Это неизбежно. Например, я каждый день ставлю перед собой цели, сколько я хочу заниматься математикой, и обычно в итоге я занимаюсь математикой без перерыва. Тем не менее, мне это очень нравится, и я не хотел бы, чтобы это было каким-то другим способом.

Но могу дать один небольшой совет: постарайтесь вставать пораньше , допустим, в 6 утра. Тем не менее, убедитесь, что вы спите не менее 8 часов; поэтому ложитесь спать в 21:00. Сон – один из самых важных моментов, когда дело доходит до учебы. За много лет занятий математикой я обнаружил, что наиболее продуктивн и энергичен до 12:00 . Если вы сможете закончить большую часть своей работы до 12:00, то у вас будет действительно хорошая возможность хорошо справляться каждый день. Кроме того, старайтесь не есть большими порциями . Обильные приемы пищи часто заставляют вас терять концентрацию, а это, в свою очередь, может привести к потере нескольких часов впустую.

Я думаю, что самый важный момент, когда вы намереваетесь достичь какой-либо цели в своей жизни, — это идти к ней день за днем, час за часом, даже минуту за минутой . Часто вы можете слишком усложнить цели, думая о том, что вы хотели бы сделать в течение следующего 1 года или даже одного месяца. Если вы будете усердно работать каждый день и ставить перед собой реалистичные цели, тогда все станет возможным .

Я надеюсь, что я помог! (Я надеюсь, что мое использование полужирного текста не считается оскорбительным; я просто использовал его, чтобы выделить некоторые ключевые моменты в своем ответе.)

Отказ от ответственности (25 декабря 2013 г.): этот ответ был написан, когда мне было 16 лет, и не обязательно отражает мои текущие взгляды на математику. (Некоторые пункты, например, «запишите всю математику, которую вы изучаете», я бы сегодня никому не рекомендовал.) Но я оставлю свой ответ здесь, потому что я думаю, что это в целом разумный совет, и он явно был полезен многим людям, поскольку Об этом свидетельствуют 77 голосов.

Жирный текст всегда приветствуется, чтобы выделить важные части. Я прошел этап обучения тому, как читать математический текст достаточно строго и задавать себе вопросы. Проблема в том, что я задаю себе слишком много вопросов и трачу слишком много времени на то, чтобы что-то понять. Я спрашиваю, как выучить математику быстро и тщательно.
«Лучший способ — записать всю математику, которую вы изучаете. (…) Запишите каждое определение, каждую теорему и каждое доказательство». Это несколько раз приходило мне в голову, и я пытался это сделать. Я согласен, что это один из лучших способов усвоить и запомнить материал, при условии, что вещи, которые вы записываете, написаны вашими словами и мыслями. Но, конечно, большая проблема, опять же: время.
@wildildildlife Я в некоторой степени не согласен с этим. Это правда, что запись всей математики, которую вы изучаете, требует времени, но я считаю, что требуется еще больше времени, чтобы заново выучить математику, которую вы забыли и не записали. Например, я изучал теорию конечных групп три раза, потому что каждый раз забывал материал. (Конечно, для «переучивания» материала требуется меньше времени, потому что вы его уже видели, но общее количество времени, необходимое для закрепления материала в вашей долговременной памяти, кажется намного больше, чем общее количество времени, которое требуется для прочтения. запишите математику)
Ну, я все еще согласен с вами. Но это факт, что во время моих курсов у меня нет на это времени. Выполнение всех упражнений, которые я должен делать, и чтение, чтобы иметь возможность выполнять их, занимает почти все время. Остальное заполнено попытками построить и прочитать доказательства. Чтобы напечатать все это в понятной форме, потребуется дополнительно 3 дня в неделю. Возможно, живые текстовые сообщения (во время лекций) помогут. В противном случае мне пришлось бы сделать это во время моего драгоценного летнего отпуска.
Или, возможно, мы должны закрыть math.stachexhange, это освободит немного времени :)
Я думаю, что математику не следует изучать, поскольку вы говорите: «Вы изучали теорию конечных групп три раза». В исследованиях лучше знать, где найти результаты, чем знать каждый результат. Кроме того, я думаю, что пытаться доказать каждую теорему, с которой вы сталкиваетесь, самостоятельно — это что-то вроде излишества. Никто не может перепроверить все, что было написано ранее. Попробуйте Эванса и Гарипи, Теорию меры и тонкие свойства функций; большинство доказательств занимают страницы. Конечно, иногда после долгого чтения корректура приходит сама собой, но не при первом чтении.
@BeniBogosel Я согласен с тобой. Я понимаю, что держать в уме все математические результаты в определенном сборнике учебников довольно бессмысленно, учитывая, что даже это будет лишь небольшой долей текущих математических знаний и само по себе не поможет сделать то, что никто никогда не делал. до. Кроме того, даже если вы не доказываете каждую теорему самостоятельно, вы все равно можете разбить каждую теорему на шаги (или абзацы) и доказать каждый шаг (или абзац) самостоятельно. Я думаю, что это более эффективно, а также помогает очень хорошо понять доказательство.
«...лучший способ не слишком беспокоиться о том, что вы забудете математику, — это решить ее самостоятельно». Определенно. Трудно запомнить вид с вершины горы, если ты видел его только на картинке; это легко, если вы поднялись на гору, чтобы увидеть это.
@AmiteshDatta, когда вы говорите: «На самом деле, я записал 600 страниц математики, когда я прочитал только 315 страниц!», Тогда не будет ли перечитывание ваших заметок тратить больше времени, чем перечитывание самой книги?
Привет @Pacerier, спасибо за ваш комментарий. На самом деле я никогда не возвращался и не перечитывал свои заметки (хотя в какой-то момент мне было бы любопытно сделать это, хотя бы просто посмотреть, о чем я думал, когда впервые изучал предмет). Если я делаю обзор предмета, то по причине, которую вы упомянули, я бы вернулся и перечитал соответствующие части книги (в любом случае, это, вероятно, было бы лучше, чем мои черновые заметки). В настоящее время я не принимаю этот стиль обучения; это занимает слишком много времени и не обязательно помогает мне лучше запомнить/понять тему!
@AmiteshDatta, ты все еще хранишь эти 600 страниц заметок? Они в мягкой копии?
Привет @Pacerier, я написал их в нескольких блокнотах от руки, и они, вероятно, где-то в моем доме. Возможно, я попытаюсь найти их теперь, когда вы упомянули об этом!
@AmiteshDatta Я только начинаю самостоятельно изучать реальный анализ. Я буду следовать вашему совету, например, полчаса подумать над определением, вывести основные свойства, доказать теорему, не видя ее доказательства. Как вы думаете, этот подход для среднего новичка? Спасибо
Привет @JPG, спасибо за ваш комментарий! Мне сложно что-либо рекомендовать, не зная вашей конкретной ситуации, но я думаю, что главное — думать о математике по-своему, помимо того, что вы изучаете. Конечно, есть много способов сделать это, и я предложил только один из них. Стоит попробовать различные подходы, а затем решить, какие из них работают для вас.
@AmiteshDatta, но время - ограниченный ресурс. Я думаю, что он пытается сэкономить время, учась на ваших ошибках. Он спрашивает, какие вещи, которые вы сделали, которые вы нашли, являются пожирателями времени? В общем, какие ошибки вы сделали?
Привет @Pacerier, спасибо за ваш комментарий; неплохо подмечено. Я думаю, будет справедливо сказать, что это был не лучший подход для меня, чтобы делать так много заметок, оглядываясь назад. Это не помогло мне лучше запомнить математику; Я думаю, что лучше не слишком беспокоиться о том, чтобы что-то забыть, и просто смириться с этим. По мере того, как человек становится более математически опытным, действительно важные идеи становятся ясными, а также более широкое (более) видение того, что касается определенных аспектов предмета (подробности доказательств и т. д. иногда лучше забыть) ...
Кроме того, доказательство результатов самостоятельно, безусловно, помогает активно участвовать в предмете и является отличным способом (я думаю) проверить свое понимание на раннем этапе (для меня это был небольшой скачок, когда я впервые начал читать серьезные учебники по математике после математических вычислений). , потому что решений упражнений не было, поэтому я изначально начал воспринимать теоремы как упражнения с решениями). Однако в долгосрочной перспективе может потребоваться много времени, чтобы доказать все (или даже многие нетривиальные вещи) самостоятельно; это сложный баланс. (Кроме того, некоторые вещи кажутся тривиальными, если у вас правильная точка зрения.)
В конечном счете, то, как человек изучает математику, действительно зависит от многих факторов, таких как его опыт, насколько он математически зрел, сколько времени у него есть, как он может изучать новые вещи и т. д., поэтому трудно дать действительно конкретный ответ. . Я имею в виду, что одна вещь, которая хорошо работала для меня на раннем этапе, заключалась в том, чтобы не обязательно изучать вещи по порядку, а скорее по мере того, как я видел интересное. Например, я изучал топологию (точечную и базовую алгебраическую топологию), не изучив по-настоящему линейную алгебру, реальный анализ и комплексный анализ; Я как бы просто искал некоторые вещи в реальном анализе по мере необходимости...
В конце концов, мне удалось охватить основные темы, и это меня устраивало лучше, чем «систематическое» изучение математики. В принципе, я бы не рекомендовал это начинающему студенту-математику (на самом деле, я могу даже не рекомендовать его более молодой версии себя), но я так учился, по крайней мере, раньше. Главный совет, который я могу честно дать, — проявить творческий подход и найти свой собственный путь в математике, а затем давать советы другим людям на основе того, что вы узнали! Иногда единственный способ научиться на некоторых ошибках — это совершить их самостоятельно.
Привет @JPG, пожалуйста, ознакомьтесь с моими четырьмя комментариями выше, где я даю более подробный ответ на ваш вопрос.
@AmiteshDatta Хм, спасибо
@AmiteshDatta Итак, тебе было 16, и ты занимался теорией групп? Мужчина...
Привет, @AmiteshDatta, я только сейчас увидел этот пост. Когда вы сказали «день за днем, час за часом...» в последних нескольких строках, означает ли это, что вы занимались математикой, просто проснувшись и сосредоточившись на попытке понять какую-то концепцию, например, и проснувшись день и заметили, что вы закончили 10 или около того продвинутых тем? Я спрашиваю, потому что мне часто кажется, что я могу понять, над чем сейчас работаю, но трудно понять долгосрочную выгоду. Вот почему мне кажется интересным то, что вы, кажется, говорите, что никогда не заботились о долгосрочной выгоде/прибыли. Спасибо!
Привет, @johnfowles, спасибо за комментарий! Да, я думаю, что каждый день я фокусировался на какой-то конкретной цели, которую считал достижимой (но только подталкивая себя). Я думаю, что одна из вещей, которая помогла мне мотивировать, заключалась в том, чтобы иметь в виду долгосрочную цель (какой-то большой проект, например, когда я учился, пытаясь прочитать всю книгу по математике) и составить план на день за днем. помогите мне достичь этой цели. Я знал, что если я пропущу день, мне будет очень трудно наверстать упущенное, поэтому я должен был усердно работать каждый день, и у меня всегда была долгосрочная цель.
"допустим, в 6:00 утра. Однако убедитесь, что вы спите не менее 8 часов, поэтому ложитесь спать в 9:00 вечера" . 9-6 это 9 часов ;)
@AmiteshDatta Вы изучали теорию групп в 16 лет? !!!

Большинству людей этот ответ, вероятно, не понравится, но математика — это область, в которой существует нестабильное разделение между теми, кто обладает гениальным пониманием, и теми, кто просто способен выжить благодаря упорной тяжелой работе. Слишком много людей хотят делать доказательства и эстетически приятную искусную математику для карьеры, которые находятся в категории упрямого упорного труда. Я говорю как человек, который работал до смерти за последние 3 года, чтобы взломать его в докторской степени Лиги плюща. программа по прикладной математике. По сравнению с моими сверстниками, единственное преимущество, которое у меня есть, это то, что я могу работать намного усерднее, и в какой-то степени я намного лучше пишу программы. Что касается математических способностей, то все они доминируют надо мной.

Если, как вы признаете, вы средний человек с плохой памятью (если не считать вашей явно выше средней терпимости к сложной технической работе), то вам следует учитывать, что реальная карьера в чистой математике вам не подходит. Я хочу быть осторожным, чтобы избежать оптимизации других, поэтому, пожалуйста, примите мой совет с долей скептицизма. Возможно, это не подходит для вас, и, конечно же, у всех других комментаторов тоже есть дельные советы. Но одна вещь, которая, я думаю, никогда не сработает для вас, это просто «стараться заниматься спортом, правильно питаться и вести сбалансированную жизнь». Что бы ни говорили другие, этого не произойдет с чистым математиком, у которого действительно хороший вкус в эстетической красоте результатов, если только этот математик действительно не находится на уровне гения.

У вас ограниченное количество талантов и ограниченный бюджет времени. Ваш личный прогноз о том, что вам понравится заниматься абстрактной математикой, почти наверняка неверен; вы, кажется, недооцениваете такие важные вещи, как зарплата, конкурентоспособность на должности, географические предпочтения и т. д.

Например, у меня есть близкий друг, который изучал очень чистые аспекты криптографической теории чисел. Он сделал два постдока и практически не заработал денег, пожертвовал личными отношениями, чтобы попытаться получить постоянную должность на факультете, и в конечном итоге не нашел работы, связанной с чистой математикой. Он устроился программистом в компанию, производящую криптографическое программное обеспечение. Он думал, что по крайней мере часть его времени уйдет на исследование новых абстрактных идей в криптографии, но это оказалось не так. Вместо этого большую часть времени он пишет программы на Java, узнает о новых прикладных исследованиях в области криптографии и очень мало пишет (хотя в личное время он по-прежнему занимается исследованиями и, на мой взгляд, гораздо лучше осведомлен о криптографических исследованиях, чем большинство людей, которые в настоящее время занимаются исследованиями в области криптографии). опубликовать в этом поле).

Он несчастлив в этой ситуации? Нет! На самом деле, он обнаружил, что для правильного проектирования программного обеспечения практически все зависит от понимания правильной абстракции, правильной инкапсуляции данных, правильного шаблона проектирования, и это не только имеет большую математическую эстетическую ценность, но и обеспечивает лучший продукт для клиента. . Привыкнув к профессиональной разработке программного обеспечения, он теперь видит всевозможные параллели между своей прежней работой, связанной с абстрактными математическими идеями, и своей нынешней работой, связанной с абстрактными программными решениями. Его набор навыков теперь имеет гораздо более высокий экономический спрос, его не заставляют конкурировать за штатные должности, и он может поддерживать очень здоровый баланс между работой и личной жизнью благодаря регулярному графику работы своей компании.

Я бы сказал, что так же, как многие малые предприятия терпят неудачу, слишком много блестящих аспирантов видят себя следующим Гёделем, фанатично занимающим штатные должности и «живущим жизнью разума». Они особенно склонны делать то же, что и вы, и позволять остальной части своей жизни ухудшаться в надежде на то, что они в настоящее время (возможно, ошибочно) считают своим собственным предпочтением абстрактной красоты, которую можно только насытить (также ошибка) по обобщенной математике. Многие из этих людей должны признать тот факт, что они недостаточно талантливы, и что университеты, которым приходится выделять все меньше ресурсов для найма лучших штатных преподавателей, не должны их нанимать.

Вот несколько ссылок для размышления:

Экономика тоже имеет значение. Срок пребывания в должности значительно сокращается во многих академических кругах, и математические факультеты особенно печально известны, потому что чистая математика не приносит грантов, как прикладные проекты. С появлением онлайн-курсов и открытых учебных программ, а также таких сайтов, как Stack Exchange, потребность в высокоспециализированных учителях математики на университетском уровне уменьшается. Вы должны ожидать, что конкуренция за постоянную работу ужесточится, и если вы хотите постоянную работу, вам придется идти туда, где ее предлагают, даже если это небольшой региональный университет, который находится далеко от любого крупного города, не имеет настоящей культурной атмосферы. , и не привлекает одаренных студентов. Не учитывать это было бы большой ошибкой.

Мой вам совет такой. Подумайте хорошенько о том, что конкретно вам нравится в математике. Если вам нравятся абстракции и геометрическое мышление, которые часто являются частью продвинутого анализа и топологии, то существует множество карьерных путей в прикладной математике/прикладной физике/инженерии, которые дадут вам возможность изучить математические вопросы, но также привнесут способность к геометрической абстракции. работать над написанием программного обеспечения для решения актуальных проблем. Ваше знакомство с чистой математикой может дать вам преимущество в карьере, если вы переключитесь на подобную область. Вы можете более эффективно конкурировать за гранты и должности преподавателей, если это то, чего вы хотите, и в той мере, в какой вы освоите навыки программирования, у вас будут рыночные навыки, чтобы получить другую работу, если возникнет необходимость.

Если вы предпочитаете более абстрактное мышление, которое часто сопровождает алгебру и теорию чисел (то есть, если вы являетесь чистым математиком типа «решателя задач» в соответствии с определением Тимоти Гауэрса , то, я думаю, вы найдете много интересного в разработке программного обеспечения). Возможно, вам будет лучше сосредоточиться на абстрактных проблемах информатики и разработки программного обеспечения.

Если вы читаете хорошую книгу по истории математики (например, Stillwell), вы заметите, что (а) большая часть хорошей абстрактной математики начинается с того, что она является своего рода специальной интуицией, «это, вероятно, правда, но я не могу видеть детали», и только позже она уточняется; и (б) самые удивительные вещи, изобретенные в математике, не были изобретены людьми, которые думали, что математика — это способ, которым они должны зарабатывать на жизнь. Люди тысячелетиями сводили себя с ума, решая математические задачи, не ложась спать до поздней ночи, ведя разрушительную романтическую жизнь, впадая в болезни. Если вы действительно любите математику, вы никогда не будете счастливы заниматься ею наполовину, как того требует здоровый баланс между работой и личной жизнью, и очень немногие люди действительно способны поддерживать подобную карьеру. Большинство в конце концов перестают усердно заниматься математикой и становятся неудовлетворенными своей карьерой.

Зарабатывать на жизнь, будучи чистым математиком, — это очень современная концепция, которая возникла в основном из-за последствий интеграции Лебега в анализе и теории вычислимости в информатике. И теперь, когда у нас достаточно информации об этих полях и их последующих дочерних элементах, просто не хватает материала, чтобы поддержать многих профессиональных математиков. Почти наверняка значительные математические достижения в следующие 50 лет будут достигнуты высокоинтеллектуальными, преданными своему делу любителями, которые решают задачи в таких местах, как Stack Exchange или полимат .

И нет никаких причин, по которым вы не можете найти некоторые нишевые проблемы, над которыми вам нравится работать, делать это в свободное время, и в то же время иметь полноценную и экономически разумную карьеру, которая обеспечивает вам более комфортную жизнь. Ибо, как бы вы ни были умны, было бы неразумно не подумать о таких вещах в юности. Многие другие студенты-математики должны сделать то же самое.

На самом деле вопиюще несправедливое недофинансирование студентов и раздувание постдокторских должностей в значительной степени происходит из-за наивных молодых людей, которые думают, что они автоматически получат должность, если просто будут усердно стараться, и которые думают, что занудная любовь к эстетической науке — это хорошо, чтобы основывают выбор карьеры на том, что, кажется, безоговорочно соглашаются на низкооплачиваемые и недостаточно застрахованные академические должности без вопросов. Поверь мне, ты не хочешь быть просто еще одним из этих людей.

Это единственный хороший совет здесь
Вроде основано на личном опыте
Некоторые из них. Но связанные статьи об отсутствии надбавки к заработной плате для обладателей докторской степени в области компьютерных наук и математики по сравнению с обладателями степени магистра, а также о росте числа постдоков в качестве главных исследователей (следовательно, большая концентрация академической занятости на более низких уровнях и меньшая на уровне младшего специалиста). или должности штатных профессоров) получены из внешних источников, основанных на рецензируемых исследованиях. Другие наблюдения, такие как ограниченный бюджет времени и необходимость делать что-то экономически продуктивное, являются более или менее стандартными атрибутами природы. Так что, если человек заботится об этом, это менее анекдотично.
Миру нужно больше математиков, работающих в других областях, помимо математики.
Как программист, изучающий математику. Я не уверен, что я чувствую по этому поводу.
Можете ли вы уточнить?
«Он недоволен этой ситуацией? Нет! На самом деле, он обнаружил, что для правильного проектирования программного обеспечения практически все зависит от понимания правильной абстракции, правильной инкапсуляции данных, правильного шаблона проектирования, и это имеет не только большую математическую эстетическую ценность. , но и поставляет клиенту более качественный продукт" - нет, это просто неправда. «Проектирование программного обеспечения» не идет ни в какое сравнение с математическими исследованиями и гораздо ближе к монотонному ручному труду. Это может доставлять удовольствие, как хороший плотник любит делать красивую мебель, но это не настоящий интеллектуальный вызов.
@Mr.F (на всякий случай, если кто-то прочтет это), я думаю, что знаю достаточно о «программной инженерии», чтобы еще раз заявить о своем мнении, что это не интеллектуальная задача, которая приближается к математическим исследованиям.
@Mr.F: На мой взгляд, Джон Донн прав в том смысле, что то, что он говорит, точно описывает разработку программного обеспечения в целом. Всегда будут исключения, когда чистота и элегантность высоко ценятся, но большую часть времени разработка программного обеспечения существует, чтобы накормить долларовых глаз. Многие известные компании постоянно выпускают программное обеспечение как можно раньше, чтобы получить деньги, даже несмотря на то, что уже известно, что в нем присутствуют тысячи ошибок. Индустрия почти полностью управляется деньгами.
@Mr.F: Да, я действительно имею в виду, что это применимо везде. Так грустно, но правда? Кроме того, я также не выступаю за использование только чистой математики, в которой абсолютно нет уверенности, что она будет иметь отношение к решению проблем в реальном мире. Во всяком случае, только мое мнение. "="
Я могу неправильно прочитать этот пост, но, похоже, есть предположение, что не стоит заниматься академической карьерой в области чистой математики, если вы посредственны или второсортны. Не знаю, как я отношусь к этому предложению. Также нет упоминания о другой части профессии профессора, а именно о преподавании. Это упущение вызывает у меня беспокойство: есть причины, помимо исследований , рассматривать академию.
@JesseMadnick Когда я был аспирантом, меня активно отговаривали от преподавания. Когда я потратил дополнительное время на подготовку конспектов и интерактивных примеров для курса вероятностей, в котором я работал ассистентом преподавателя, мой консультант фактически отвел меня в сторону и предупредил, что я не трачу больше времени на исследования. Это была та же самая история, которую я слышал от своих сверстников, а также от моих многочисленных контактов и друзей в академических кругах других университетов и во многих различных дисциплинах (не только математике или естественных науках).
Когда вы объединяете это со статьями, такими как The Economist «The Disposable Academic», и вы смотрите на статистику о темпах роста вспомогательного персонала, который, как ожидается, возьмет на себя основную часть преподавательских обязанностей, при этом ему платят гораздо меньше, учитывая менее всеобъемлющие пакеты льгот. , и почти нулевые возможности для карьерного роста, я действительно думаю, что очень неразумно заниматься наукой ради того, чтобы попасть на университетский уровень преподавания. Это почти как лотерея. Возможно, 1/1000 лучших выпускников аспирантов сделают успешную преподавательскую карьеру. Остальные публикуются или погибают и идут в промышленность.
@EMS, что именно означает «нестабильное разделение»?
Здесь я вскользь имею в виду, что никто не "близок" к границе. Если вы достаточно хорошо разбираетесь в математике, так что выполнение очень сложной профессиональной математики не мешает вам иметь хороший баланс между работой и личной жизнью, тогда ваши математические навыки намного, намного выше, чем даже у среднего доктора математических наук. окончить лучшие ВУЗы. С другой стороны, если у вас нет таких экстремальных способностей в математике, но вы по-прежнему усердно работаете настолько усердно, насколько это необходимо, чтобы сделать высшую математику для карьеры, то у вас почти наверняка нет хорошего баланса между работой и личной жизнью. Конечно, могут быть исключения; Я утверждаю, что они очень редки.
Прошу прощения за поздний удар, и я надеюсь, что это не слишком по теме, но вы верите, что этот совет также относится ко многим потенциальным физикам?
Вы хотите сказать, что плохая память не позволит вам заниматься математикой?
Простая тяжелая работа по математике не сработает.
Я должен сказать вам, что полностью изменил свое впечатление о вас после прочтения вашего поста здесь. Отличный.
Несколько опасный ответ, так как слишком много людей отказываются от математики из-за психологических проблем и математической фобии. Это правда, что всегда есть такие гении, как Абель или Галуа, но тогда, если другие области также начнут становиться элитарными, мир был бы хуже.

Несколько очень простых советов, не связанных с математикой, и извините, если я говорю как ваша мать. Если вы чувствуете, что у вас плохая память и вы не находите достаточно времени для общения, возможно, вы не находите достаточно времени, чтобы хорошо поесть. Употребление в пищу большого количества свежих фруктов, свежих овощей, свежей рыбы, оливкового масла и злаков даст вашему телу строительные блоки, чтобы сделать все возможное. В частности, известно, что жирная рыба полезна для мозга. http://www.newscientist.com/article/mg20827801.300-mental-muscle-six-ways-to-boost-your-brain.html

Я считаю, что время, которое я трачу на приготовление пищи или мытье посуды, доставляет мне удовольствие, когда я расслабляюсь, когда приходят в голову некоторые из моих лучших идей. Возможно, вы могли бы совместить это с социальным аспектом и пригласить людей на чай, если вы готовите что-то вкусное. Избегайте алкоголя, который обычно сопровождает такие ситуации, если вы собираетесь вернуться к работе после него.

Мне очень нравится этот ответ. Важность правильного питания (а не только питания из стимуляторов) невозможно переоценить. Фрукты, конечно, особенно важны, потому что мозг работает на сахаре, а во фруктах его полно. +1
Конфеты также полны сахара! (впадает в гипогликемическую кому...)

По поводу "и вести здоровый образ жизни". Ну, вы также должны научиться вовремя останавливаться.

Иногда вы понимаете, что чего-то не понимаете, и у вас нет времени, чтобы понять это. Постарайтесь «упаковать» это как можно больше. Выясните общую форму того, чего вы не понимаете. Как это работает? В каком контексте используется эта идея? Какие у него есть «входы»? Что-то появляется как по волшебству? Что? Вы когда-нибудь видели что-нибудь подобное раньше? Если вы держите в уме эти смутные идеи, вы вполне можете понять их во сне, в разговоре с кем-то, может быть, спустя недели, а может быть, и годы. Это зависит от конкретной вещи.

Иногда ты чего-то не понимаешь, потому что это полная ерунда. Профессионалы иногда говорят ерунду — они люди и ошибаются. Книги делают ошибки. Я помню, как потратил много времени, пытаясь завершить доказательство для домашней задачи, и все, что я пробовал, терпело неудачу. За час до выполнения домашнего задания я разговаривал с кем-то еще в классе. Он показал мне пример, который он придумал для демонстрации теоремы, и я переинтерпретировал его как пример, опровергающий теорему. Такие вещи случаются. Точно так же во многих учебниках есть тонкие «ленивые» ошибки. Если вы не особенно уверены в себе, вы можете часами расстраиваться из-за такой проблемы. Разговаривайте с людьми.

Вы совершенно правы насчет «узнать, когда остановиться», но когда я читаю заметки, я ставлю себя в положение, что ни одна из этих вещей не является правдой, что я тот, кто должен это проверить, а затем быть в состоянии воспроизвести доказательство для других. Я предполагаю, что это именно та работа, которую выполняют исследователи. А в исследованиях одна незамеченная ошибка может привести к тому, что вся теория развалится, как костяшки домино. Если я не могу понять большинство доказательств сейчас, когда результаты известны и представлены мне, как я смогу открыть новые?
Это помогает создавать «модули», с которыми вы чувствуете себя комфортно и уверенно. Во многих вводных классах многообразий многие доказательства сводятся к некоторому пониманию теоремы о неявной или обратной функции. Таким образом, «жестко запрограммированные» эти теоремы являются ключом к тому, чтобы сделать переход к теории многообразия более скромным шагом. Таким образом, ваша мотивация в каком-то смысле не должна доходить до того, что вы можете объяснить доказательство кому-то еще, она должна доходить до точки, когда доказательство кажется вам естественным. Во многих (большинстве) предметах есть всего несколько ключевых идей. Получение чувства для
их и сделать естественные экстраполяции является ключевым. В этом отношении я считаю домашнюю работу очень ценной.
+1 за «Если вы не особенно уверены в себе, вы можете часами расстраиваться из-за такой проблемы. Поговорите с людьми».
@LeonLampret: Каждая форма доказательства была новой в какой-то момент и была обнаружена (вероятно, несколько раз, прежде чем она прижилась) кем-то, кто не знал ее и никогда не представлял ее им. Открытие новых доказательств связано с умением, а не с «пониманием представленных мне доказательств». Это не ваша работа — воспроизводить доказательство для других, это работа публикаций. Наконец, публикация незавершенной работы действительно дает результаты, которые разваливаются, как костяшки домино. Вас выставили на протоптанные тропы. На границе часто случаются ошибки.
+1: «Постарайтесь максимально «упаковать» это».

Об изучении математики:

Ваше время на этой скале конечно, количество математических знаний бесконечно. Вы должны мудро выбирать, на что вы хотите потратить свое время, изучая. Решите, хотите ли вы быть «мастером всего, мастером ничего» или «мастером одного, мастером ничего».

Лично я предпочитаю знать много обо всем, чем все об одном, поэтому я бы посоветовал не тратить время на изучение каждой детали. Оцените материал высокого уровня, переходите к следующему полю, пока не закончатся поля. Затем, насколько позволяет время, переходите к материалу более низкого уровня.

Ваше обучение не прекращается после получения степени (степеней). Научитесь контролировать себя.

Что касается здорового образа жизни:

Ум может быть таким же острым, как и тело. Выделяйте время на физические упражнения, правильно питайтесь и высыпайтесь. Вы обнаружите, что мыслите яснее, лучше усваиваете информацию и чувствуете себя счастливее.

Баланс критически важен. У вас есть только одна жизнь, и это гораздо больше, чем математика. Потратьте время, чтобы изучить другие интересы, открыть для себя новые и стать более разносторонним. Чем больше ваши общие знания, тем лучше вы будете в математике.

Живая жизнь. Общайтесь, влюбляйтесь, пробегайте марафон, вступайте в драку, ходите на балет, рисуйте шедевры, отправляйтесь на рыбалку, исследуйте мир; делай то, что делает тебя больше, чем просто математиком, делай то, что делает тебя личностью.

Ввязываться в драку. Как Эварист Галуа , да? :-)

Я вижу, что этому вопросу уже несколько месяцев, но я хотел бы добавить несколько замечаний:

1) У большинства математиков-исследователей память лучше и они быстрее, чем вы описываете. Есть заметные исключения, но вы должны понимать, что конкурировать будет сложно. Вам всегда придется работать усерднее, чем большинству ваших сверстников. Если вам потребуется в три раза больше времени, чтобы исправить экзамены, то это время будет упущено из вашего исследования, даже если вы, возможно, столь же талантливы в реальных исследованиях. С другой стороны, не верьте своим однокурсникам, когда они просто говорят , что понимают вещи и работают быстро. Во многих местах круто заявить, что сдал экзамен, затрачивая на это мало времени. В конце концов, вы можете обогнать некоторых людей, которые умеют учиться только ради экзамена.

2) Из вашей группы сверстников очень небольшой процент станет исследователями. Нет смысла сравнивать себя с людьми, которые успешно сдают экзамены, если вы хотите стать исследователем. Ищите хороших, амбициозных студентов и общайтесь с ними. Если они сообразительнее и имеют лучшую память, чем вы, то спросите их , что не так с леммой 3.4, доказательство которой кажется вам каким-то странным. Нет смысла искать все глупые ошибки самостоятельно. Спросите своих коллег, спросите своих профессоров, спросите здесь. Вы теряете время, если вам потребуется три часа, чтобы узнать, что профессор написал «с» вместо «е».

3) Если вы слишком концентрируетесь на деталях, вам нужно тренировать резюме. Можете ли вы объяснить очень талантливому начинающему студенту, что они узнают из линейной алгебры и анализа? В 10 предложениях? Через пару минут? Через пару часов? Без бумаги? Когда мне нужен результат от лекции, которую я услышал в студенчестве 15 лет назад, мне не нужнозапомнить условия теоремы. Мне нужно осознать, что эта теорема, вероятно, применима к моей проблеме, в какой лекции или книге я ее видел, и тогда я могу посмотреть ее, чтобы проверить, не было ли какое-то техническое условие, которое я забыл. Для этого я должен помнить суть теоремы и доказательства, а не детали. Также, если вы что-то не поняли во время обучения, предварительно примите результат, продолжите и вернитесь к результату позже, не размышляйте об одном до бесконечности.

4) Обычно суть — это то, что профессора любят слышать во время устного экзамена. Они проверят детали тут или там, но им не нужно постоянно выслушивать мельчайшие детали. Вы уверены, что действительно говорите с ожидаемым уровнем детализации во время устных экзаменов? Или вы просто предполагаете, что профессор хочет выслушать все подробности и сразу начать с эпсилон-уровня? Вы когда-нибудь пытались присутствовать на экзаменах других студентов?

5) Ищите младших школьников и помогайте им готовиться к экзаменам (или отвечайте на вопросы здесь). Помощь другим — лучший способ сохранить свежесть полученных знаний. Это не будет потрачено впустую.

6) Вы ни в коем случае не должны жертвовать своим физическим и психическим здоровьем. Сон, еда, физические упражнения, социальная жизнь и хобби важны, и ими нельзя пренебрегать в течение длительного периода времени. Разве нельзя просто брать меньше лекций в семестр? Кого это будет волновать позже, если вам понадобится на год больше, чтобы закончить? Что-то должно уступать, и мне кажется, что проще всего для вас просто растянуть работу на большее количество времени. (И да, я понимаю, что даже без обучения существуют высокие альтернативные издержки, но сейчас вам, похоже, нужно больше времени.)

Быть в хорошей физической форме, делает вашу голову более ясной. Кроме того, вам достаточно всего одного часа упражнений в день, чтобы поддерживать приличную форму. Займитесь многозадачностью! Ходить и думать можно одновременно, вы тренируете память и выносливость.

Решение математических задач в уме — хорошее умственное упражнение.

У одного из моих профессоров был знакомый в аспирантуре, который довел это до крайности. Они сидели вдвоем и пили кофе, когда мой профессор начал что-то писать на салфетке, когда его приятель прервал его, сказав: «Настоящие математики не используют бумагу». Я предполагаю, что он был умным парнем и не шутил! Конечно, это второстепенный случай. Удачи в учебе ОП
также см. [BDNF][1] [1]: ncbi.nlm.nih.gov/pubmed/21198979
Я определенно согласен +1. Однако для OP я рекомендую сыграть в карточную игру Krypto с номерами домов.

Если вы решили изучать математику так усердно, как вы это описываете, то это значит, что она должна вам нравиться, и я думаю, вам это нравится. Тем не менее, говоря, что это съедает все ваше время, меня беспокоит. Математика должна доставлять удовольствие; по крайней мере, это так для меня. Математика не должна занимать все ваше время, потому что ваш мозг должен отдыхать, чтобы вам было легче обрабатывать вещи. Вот несколько советов:

  • при изучении выберите степень детализации или глубины, которую вы хотите пройти. Не вырабатывайте все доказательства из книги, изучая ее. Выбирайте то, что вас действительно интересует и что вам нужно для вашего курса. Обычно курс не охватывает всю книгу, и для получения хороших оценок вам не нужно больше, чем то, чему вас учили в курсе. Некоторые вещи станут ясны только со временем и опытом; вы выучите их к экзамену, но всю картину поймете через больший промежуток времени, может быть, годы.

  • научиться расслабляться каждый день. Может я и ленивая, но всегда нахожу время на прогулку, на велосипедную прогулку, на игру на фортепиано, на просмотр фильма. Например, я расслабляюсь при решении задач на этом сайте, или самостоятельно, задач, которые не имеют ничего общего с тем, что я изучаю в данный момент. Спите не менее 8 часов в сутки. Расслабляясь, вы оставляете мозгу возможность привести все в порядок. У многих математиков были откровенные идеи, когда они занимались обычными делами. Прогулка по парку может помочь вам понять ключевой момент в доказательстве, или решение проблемы может всплыть, когда вы занимаетесь каким-то спортом или какими-то делами по дому (это случалось со мной не раз; самое смешное, что я решил задачу, поставленную перед тестом по отбору команды для ИМО в моей голове,

  • обычно вы можете лучше сосредоточиться, если у вас есть более важная цель, чем «закончить книгу». Например, возьмите статью из той области, которую вы изучаете (возможно, с этим вам поможет преподаватель) и постарайтесь понять ее в деталях. Изучайте только те теоремы и доказательства, которые относятся к этому. За последние годы математика сильно развилась. Пытаться успевать за всем невозможно. Сосредоточиться на более узкой области обычно проще, а в исследованиях это действительно необходимо.

  • никогда не беспокойтесь о запоминании всего. Вы забудете многое, сколько бы раз вы их ни учили, но самое главное — помнить, где искать то, что вы забыли. Например: теорема X с примерами и контрпримерами представлена ​​в книге Y, предмет Z можно найти в книге T и так далее. Попробуйте разбить доказательства на этапы, которые вы сможете запомнить. Не запоминайте расчеты. Запомните только ключевые моменты и поверьте себе, что сможете заполнить пробелы.

  • найдите время, чтобы провести его с друзьями или коллегами. Наличие кого-то, с кем можно поделиться идеей, даже математической, может оказаться очень полезным.

  • найдите кого-нибудь, кого вы можете обучать (на уровне старшей школы или университета, на первом курсе). Это может оказать большую финансовую помощь, и вы заметите, насколько хорошо вы понимаете вещи, когда пытаетесь объяснить их тому, кто их вообще не знает. Это очень помогло мне.

Удачи.

Я студент американского университета, переживающий почти то же самое, что вы описываете: недостаток сна, недостаток общественной жизни, слабая память на доказательства и перфекционистское требование, чтобы математика представлялась «по-моему». Как я уже сказал, я в настоящее время прохожу через это, поэтому я не могу дать никаких ответов. Однако у меня есть несколько советов, основанных на опыте.


Недавно я оказался в неприятной ситуации, когда за день до экзамена мне пришлось запоминать множество корректур. Внезапно уже не имело значения, знал ли я лишь несколько доказательств в мельчайших подробностях. Что мне было нужно, так это знать все доказательства, но только настолько подробно, чтобы гарантировать достаточное частичное доверие. Для этого я просматривал доказательства в своем учебнике одно за другим, записывая небольшие резюме каждого из них своими словами.

Я хочу сказать, что это (для меня) было очень эффективным методом уловить основные идеи доказательств, не зацикливаясь на деталях. При написании собственных резюме я также смог свести целые доказательства к паре предложений, которые затем служили мнемоникой для запоминания.

Но что касается вопросов, которые вы на самом деле задали...

Стоит ли пропускать корректуру? В идеале, вы бы прочитали и поняли их все, но если у вас мало времени (а вы, кажется, это делаете), то вы должны быть эффективными. Райан Бадни прав: нужно научиться вовремя останавливаться. Узнайте, что, по вашему мнению, имеет отношение к успешной учебе в классе. Затем, когда курс закончится, вы можете потратить время на то, чтобы разобраться в деталях или менее важных доказательствах или во всем, что вы хотите, если вы того пожелаете.

Должны ли вы стараться меньше, получать худшие оценки, но «жить жизнью»? Я не думаю, что кто-то может ответить на этот вопрос, кроме вас, я боюсь.

Я скажу, однако, что эффективность действительно имеет значение, и что вы могли бы найти способы сбалансировать учебу с общественной жизнью, если вы их ищете. Знаешь, почему-то мы все довольно эффективны, когда приходит время экзамена, умудряясь втиснуть большое количество информации за очень короткий промежуток времени. У нас нет другого выбора, кроме как быть эффективными. Поэтому, хотя я не говорю, что вы должны относиться к каждому дню так, как будто это день перед экзаменом, я думаю, что вы можете найти способы повышения эффективности, если будете их искать.


Я должен указать, что все это предназначено, чтобы быть практическим советом, а не мудрым советом. Чтобы получить мудрый совет, я также рекомендую советы Терренса Тао по вопросам карьеры, а также общение с вашими профессорами и советниками.

Наконец, я должен упомянуть, что, насколько я понимаю, — хотя я еще ни в коем случае не профессиональный математик — в конце концов, дисциплина и трудолюбие имеют такое же значение, как и природный талант, если не больше.

Так что, если вы беспокоитесь о том, сможете ли вы производить математику исследовательского уровня, то мой совет — перестаньте об этом беспокоиться. Если вы еще не пробовали свои силы в исследованиях, то нет причин беспокоиться об этом преждевременно. По крайней мере, это то, что сказал мне мой консультант, когда я рассказал ему об этих опасениях в прошлом году, и действительно, это был один из лучших советов, которые я когда-либо получал.

Среди советов по карьере Терренса Тао я обнаружил, что его описание до-строгих, строгих и пост-строгих стадий математического образования особенно важно для меня. Это звучит актуально и для вас: terrytao.wordpress.com/career-advice/…
И я тоже убедился, что это правда. В последнее время я предпринимаю сознательные усилия, чтобы покинуть «строгую» стадию и перейти на «постстрогую». И я должен сказать — опять же, как упоминает Райан Бадни — я начинаю видеть определенные закономерности в доказательствах. Я думаю, что такое распознавание образов рано или поздно случается с большинством людей, но сначала они должны получить достаточно материала!
Очень интересно, рад, что я не одинок в этом, а также дельный совет. Последние пару дней я задавался вопросом, если бы я каким-то образом (не смейтесь, это серьезно) навсегда перенесся назад во времени к древним грекам, сколько бы я смог их научить математике. Особенно, если они не «покупали» мои доказательства. Я был бы разочарован, если бы узнал...

Друг прислал ссылку на этот пост, и я нахожу его очень интересным. Особенно интересна ваша убежденность в том, что у вас нет таланта. Как узнать, есть у тебя талант или нет? Что вообще такое талант? Почему вы так уверены, что ваши коллеги, которые быстрее получают доказательство или дольше его помнят, более талантливы, чем вы?

Очень сложно объяснить, что такое математический талант. На решение и доказательство большинства математических задач, которые стоит решить, и большинства теорем, которые стоит доказать, уходят годы, поэтому скорость или память не пригодятся при решении или доказательстве этих теорем. Я считаю, что у большинства людей достаточно памяти, чтобы хранить всю необходимую информацию, чтобы работать над важными проблемами в течение нескольких лет (у них достаточно времени для этого). Скорость в трех- или четырехлетнем проекте почти никогда не бывает полезной (конечно, есть заметные исключения, но в среднем я бы сказал, что это не имеет значения). Что на самом деле гораздо полезнее, так это трудолюбие, которое, кажется, у вас есть. Если вы можете оставаться с проблемой после нескольких месяцев неудач, то, я думаю, у вас есть нужные качества.

Одна вещь, которую люди часто поднимают, — это воображение. Этого вы никогда не узнаете, есть у вас или нет, пока вы действительно не начнете работать над проблемами. Вы можете быстро выучить язык, но после этого вы никогда не станете поэтом. Математика старшекурсников, на самом деле большая часть математики, преподаваемая в стандартных классах, просто разрабатывает язык, который некоторые люди изучают быстрее, чем другие, и некоторые помнят больше, чем другие, но то, что они будут с ним делать, не является тем, что преподается в этих классах.

Я бы сказал, что математический талант и есть это воображение. Сказать, что кто-то математически одарен, значит просто сказать, что этот человек видит больше математических связей и отношений между математическими объектами, чем большинство людей. Математически одаренный человек обладает сильной интуицией относительно того, какое направление мысли приведет к новым красивым теориям и новым открытиям. Хорошая память и скорость выполнения математических вычислений и логических операций часто ошибочно принимаются за математический талант. Это, конечно, талант, очень полезный, но я бы не назвал его математическим талантом.

В любом случае, вы, кажется, далеки от того места в своей жизни, где вы действительно можете понять, одарен ли вы математически или нет. Вы можете ускорить свой путь к этому, участвуя в исследовательских проектах, а не посещая математические курсы, которые являются наиболее обманчивыми индикаторами математического таланта. Вы также можете пройти курсы чтения и записаться на курсы более высокого уровня, например, курсы для выпускников. Есть также летние исследовательские программы, на которые вы можете подписаться.

Насчет здорового образа жизни не знаю. Я действительно думаю о математике, когда делаю упражнение. Это отличная вещь в профессии, но вы должны научить чему-то себя. Например, вы можете научить себя думать о математике, когда занимаетесь рутинными повседневными делами, такими как мытье посуды. Но мне потребовалось некоторое время, чтобы прийти к такому состоянию ума, поскольку, будучи студентом колледжа, я тоже вел нездоровый образ жизни и мало занимался спортом, думая, что это пустая трата времени. Но это не так! Вы можете научить себя думать о математике, делая это, а также это способ омолодить себя и справиться со стрессом.

Социальная часть жизни трудна. Вам как математику-исследователю нужно значительное количество времени для себя. Я сомневаюсь, что есть математики-исследователи с огромным кругом общения. Но приложив некоторые усилия, вы можете иметь вокруг себя достаточно людей, и эти люди, в большинстве случаев, которых я знаю, обычно очень интересные, умные и мотивирующие люди.

Что касается того, как выучить математику, ну, вы никогда не знаете. Мы все, я думаю, уделяем большое внимание изучению доказательств и их деталей, но когда мы переходим на более высокий уровень, мы понимаем, что на самом деле не поняли доказательство. Так что в некотором смысле на самом деле бессмысленно изучать доказательство теоремы, которую вы видели впервые. Вероятно, делать что-то вроде кругового цикла, когда вы изучаете вещи, и по мере того, как вы продолжаете, вы возвращаетесь к более ранним вещам, и вы изучаете их заново, это лучше.

Но я думаю, что более важно для вас понять, почему вы хотите так хорошо выучить эти доказательства. Есть еще много доказательств, которые нужно выучить, и вы никогда не будете знать их все наизусть. На самом деле, что вы хотите получить от математики? Лучше всего сосредоточиться на том, чтобы найти ту область математики, которая вам нравится, и изучать этот предмет с хорошим профессором по кругу, когда предмет становится все более и более сложным, а важные идеи, которые были оставлены позади, время от времени возвращаются.

Короче говоря, бояться нечего, и я уверен, что вы разберетесь в этих вещах сами.

Я думаю, что совет Теренса Тао по карьере может ответить на ваш вопрос. Я настоятельно рекомендую вам прочитать ее.

Редактировать: А также книгу Кевина Хьюстона «Как думать как математик: помощник по математике для студентов» .

хм, не нашел ничего, что имело бы отношение к моей актуальной проблеме: как учиться, чтобы я понимал математику, занимался досугом и заканчивал вовремя. Потому что до сих пор это было одно большое страдание и тяжелая работа...

Некоторое время назад я проходил (и до сих пор в какой-то степени) похожую фазу. Я не особенно силен во всех основных предметных областях математики и сам точен в деталях, и бывают моменты, когда все это меня немного ошеломляет. В такие моменты я либо предаюсь забавной математике, либо просто пытаюсь ради забавы доказать результаты, которые меня привлекают, независимо от того, сколько времени это займет или насколько тривиальными они кажутся. Это привязывает меня к математике и в то же время расслабляет. Кроме того, я каждый день уделяю время нематематическим занятиям, потому что, если я этого не сделаю, слишком много умственной деятельности неизменно вызывает у меня головную боль.

Я думаю, что заниматься математикой очень похоже на игру музыки. Это тяжелая работа, но иногда вы можете играть любые расслабляющие мелодии. Все сказано и сделано, я думаю, важно помнить, почему именно мы занимаемся математикой: потому что это весело!

Добавлено после просмотра комментария:

Я понимаю. Я могу как-то связать это с моими выпускными днями. Я прошел их каким-то образом с большим беспокойством, и из этого я вынес следующее: важно, чтобы вы изучали правильный путь, а этот правильный путь уникален для каждого. Помню, во втором семестре я очень мучился с «Комплексным анализом» Альфорса (я до сих пор немного опасаюсь его), и причина была в том, что эта книга не была приспособлена к моему стилю обучения, и не было времени кропотливо аргументировать все. «предполагается, что это ясно» в книге. Позже я прочитал еще одну книгу по комплексному анализу (Браун и Черчилль), и то, что я знаю об этом предмете, во многом благодаря ей. Это потому, что вторая книга была больше ориентирована на мой внутренний процесс понимания, чем Альфорс.

Спасибо за совет, но я думаю, вы не понимаете ситуации: нет времени на веселую математику, нет времени на книги или релаксацию и т. д. Каждый год у меня 12 курсов, в каждом конспекты около 100 страниц, которые я надо как-то переварить и суметь воспроизвести на экзаменах. Это просто ужасно. Я просто был бы рад «выжить» с хорошими оценками и свободным временем/физической активностью. Я знаю, что я хочу сделать, если бы у меня было время. Это курсы по математике, которые съедают все мое время. Если я не позволю этому случиться и ВЫДЕЛЯЮ немного свободного времени, последствиями будут такие, что я буду понимать еще меньше.

Я предлагаю перейти на программу, где ваши интересы максимально совпадут с предметами, которые вам необходимо изучать.

В середине первого года обучения в бакалавриате я наткнулся на программу по математике с отличием в Университете Альберты, и меня это очень зацепило. Программа отличников подчеркивала строгость, понимание, технику, визуализацию, точность, в основном просто действительно прочную основу.

На втором курсе бакалавриата у меня был (обязательный) довольно неудачный вводный курс по дифференциальным уравнениям, где все сводилось к прокручиванию формулы. Доказательств и идей нигде не было видно. Этот курс был для меня довольно разочаровывающим — казалось, что это пустая трата возможности начать связывать различные нити, которые мы разрабатывали в анализе, линейной алгебре, алгебре и так далее.

Я обнаружил, что, будучи студентом, у меня обычно было достаточно времени, чтобы сделать все, что от меня требовалось. Были моменты, когда я попадал в ситуации, которые были близки к выходу из моей головы, но все складывалось к лучшему. Если вы никогда не заставите себя слишком сильно, вы никогда не узнаете, что такое «слишком сложно». Так что это хорошо, чтобы обнаружить. Я думаю, это помогает, когда то, что ты должен делать, это то, что ты хочешь делать. Если это не так, вы можете в конечном итоге тратить время на скуку. Это хороший урок, чтобы научиться делать скучные вещи, но, надеюсь, их не слишком много в вашем бакалавриате!

Переход на другую программу будет проблематичным. В моей стране есть 2 университета, и я учусь в большем из них по программе чистой математики. Я ожидаю, что в другом университете дела обстоят еще хуже. Поездка за границу, кажется, представляет еще большие трудности. Особенно, если я хочу в конечном итоге жить здесь.
Проблема не в том, чтобы не подталкивать себя, как вы можете прочитать из моего первоначального поста. Проблема в том, что я несколько лет заставлял себя компенсировать отсутствие таланта и действительно понимать вещи, и я значительно снизил качество своей жизни. Эффективность - это то, чего мне не хватает. Я сильно подозреваю, что большинство одноклассников не читают корректуры или просто просматривают их, чтобы почувствовать. Это способ сделать это?
Почему бы не поговорить с некоторыми словенскими профессорами и не спросить их, были ли у них подобные проблемы и как они с ними справились? Вы можете поступить в университеты за пределами Словении и, в конце концов, найти работу в Словении — я знаю по крайней мере одного профессора на вашем факультете, который сделал именно это.
Будучи студентом, я не останавливался на каждом доказательстве, которое нам давали в классе. Я делал то, что требовалось в домашнем задании, и уделял много внимания тому, что казалось трудным или каким-то образом информативным на уроке — например, я помню, как моя вера в аксиому выбора пошатнулась после доказательства леммы Урысона в точечно-множественная топология. Несмотря на то, что технически доказательство имело смысл, оно все равно казалось мне неправильным .
Вы прошли все пруфы на всех курсах? У нас 12 предметов в год, у каждого в среднем 80-100 страниц (40-50 листов А4) в рукописном виде. Если многие ноты «шаткие» со множеством ошибок, как ляпсусных, так и более глубоких, как с этим справиться, не жертвуя при этом огромным количеством времени?
Будучи студентом (и до сих пор в некоторой степени), я редко делаю подробные записи в классе. Моя стратегия заключалась в том, чтобы сжать презентацию до резюме в 2 или 3 строки. Если была действительно новая идея, я подробно описывал ее, но обычно я просто пытался получить основную идею из презентации. Но у нас были учебники или распечатки конспектов лекций, так что не было особых причин делать подробные записи. Я знаю много очень успешных людей, которые все записывают и все подробно обдумывают — в этом мы все разные.
«...те, которые показались мне сложными или каким-то образом информативными в классе...» Как я могу узнать, какие корректуры информативны, не читая и не понимая их? Я часто чего-то не понимаю, потом останавливаюсь на этом, пока не прояснится. Большую часть времени это просто доказательство, но иногда я узнаю, что оно имело решающее значение для всей конструкции/доказательства. В этом был весь смысл всего этого, и если бы я этого не читал, я бы полностью упустил суть.
Я не уверен, что это полезно в краткосрочной перспективе. Но как только вы увидите «элементарные» аспекты математики в мельчайших подробностях и усвоите их, здесь я думаю об исчислении с одной переменной, начиная с аксиоматического определения действительных чисел (как в книге Апостола) и основных множеств, отношения, теория групп, основы колец, основы линейной алгебры, представление линейных отображений относительно базиса. Как только вы разберетесь с этим материалом, вам станет легче замечать очертания доказательств и мыслить скорее сверху вниз, чем снизу вверх, следуя линейно каждому шагу.
Хм, я не знаю, как выглядят теоретические (устные) экзамены в вашем университете, но здесь, на многих занятиях, в зависимости от профессора, нас просят доказать то или иное, достаточно подробно. Я усвоил студенческий материал, но, тем не менее, я не нашел никаких коротких путей / простых способов переваривать доказательства одно за другим.
О каких доказательствах вы говорите? Вы аспирант? Раньше я думал, что ты студент.
Хм, у нас есть три этапа: 1-й = 3 года (диплом), 2-й = 2 года, 3-й = 3 года (докторская степень). Я на 2-й ступени, первый курс, но учусь 5 лет, а не 4, потому что перешла с программы «прикладная математика» на программу «математика». Как я могу идти на устный экзамен, не понимая большую часть доказательств? Здесь все делают записи, и (в основном) нет распечаток конспектов лекций . Поэтому я копирую записи других, так как я не могу понять, что объясняет профессор, И правильно записать это в свой блокнот.
Да, и я говорил о серьезных (по крайней мере, с моей точки зрения) доказательствах: абстрактной алгебре, алгебраической топологии, анализе на многообразиях, теории графов и т. д.
Хорошо, я думаю, что каждый работает по-разному, когда дело доходит до таких вещей. Мне очень нравится сдавать устные экзамены, меня ставят на месте. Моя стратегия состоит в том, чтобы не слишком подробно изучать мелкие детали доказательств, а проходиться по общей схеме, иногда снова и снова, пока я сам не создам детали. Я стараюсь, насколько это возможно, избегать чтения суетливых аргументов других людей, поскольку чужое доказательство очень легко может показаться мне беспорядком. Так что, возможно, что-то вроде этого может быть стратегией — старайтесь не слишком внимательно следить за доказательствами других людей. Сделай их сам.
«старайтесь не слишком внимательно следить за доказательствами других людей. Делайте их сами». Мне очень нравится эта идея, НО она применима только к простым доказательствам. Я никак не смогу просто выяснить доказательство теоремы Зейферта ван Кампена или теоремы Гаусса-Бонне. Мне приходится читать доказательства профессора, которые иногда занимают 3 страницы, и там много непонятных мне деталей.
Поначалу SvKT может показаться неинтуитивным, но как только вы действительно его переварите, вы обнаружите, что это действительно очень просто. Это просто много «учета» и отслеживания относительно мелких деталей, которые приводят к взрыву обозначений и всех реальных технических деталей. Возможно, посмотрите на другие доказательства — у Хэтчера и Питера Мэя отличный старт. Для GB, возможно, посмотрите на доказательство Гиймена и Поллака (первоначально это принадлежит Хопфу).
пример: мы делали упражнение на курсе алгебраической топологии: вычислить π 1 диадного соленоида. У нас есть твердый тор Т в р 3 и гомеоморфизм час : р 3 р 3 такой, что час отображает тор в его внутреннюю часть, дважды свернутый. Сейчас А 0 "=" Т и А н "=" час ( Т ) . Почему я е Н А н ( А 0 А 1 А 2 ) даже непустой?? Может быть, это глупый вопрос, но я не могу сразу ответить на него. Таких вопросов никто не задает, а у меня их сотни...
«Поначалу SvKT может показаться неинтуитивным, но как только вы действительно его переварите…» Это моя точка зрения, требуется огромное количество усилий и времени, чтобы переварить большинство доказательств на моем факультете. Я подозреваю, что многие одноклассники не проходят корректуру, и у них все в порядке, но я просто не могу понять некоторые вещи, пока не потрачу достаточно времени на обдумывание и не переварю большую часть корректур. Это проблема. Я не могу понять вещи, пока не потрачу серьезное количество времени на их переваривание. Только тогда я получаю представление о техниках и методах поля. Вот почему я веду ужасный образ жизни...
Этот вопрос слишком расплывчатый, чтобы быть математическим вопросом. Например, "дважды обернутый" слишком расплывчато. Почему нет час : Т Т ?
Я знаю, что это расплывчато, но именно так мы делали это в нашем классе. Я не виню ассистента, более строгий подход наверняка привел бы к серьезным проблемам и техническим тонкостям. Тем не менее является пересечением последовательности замкнутых подмножеств А 1 А 2 А 3 р н всегда непусто?
В любом случае, я действительно не пытаюсь ни к чему придираться, но я полагаю, что при исследовании/открытии математических истин такие детали могут иметь решающее значение.
@Leon: что пересечение вложенной коллекции непустых закрытых подмножеств р н является непустым замкнутым множеством, это одна из тех фундаментальных теорем анализа, которая непосредственно следует из полноты. Это одна из тех теорем, которые интенсивно изучались на 1-м и 2-м курсах моего бакалавриата. Подумайте о последовательности Коши.
С р н полна, всякая последовательность Коши имеет предел. Лимит включен в перекрёсток, так как он закрыт. Я предполагаю, что мы выбрали точку из каждого А н чтобы получить последовательность а н . Откуда мы знаем, что это коши?
О, у меня было неправильное предположение. Наборы должны быть вложенными, непустыми и компактными . Итак, они содержатся в большой коробке. Если вы замените закрытый на компактный, вы можете взять А н быть дополнением к открытому шару радиуса н с центром в начале координат.
Да, тогда это работает. Хм, это работает и для случая диадного соленоида, так как гомеоморфный образ полнотора замкнут. Спасибо! Наверное, поэтому нам нужно час для отображения р 3 р 3 . В любом случае, таких вопросов сотни, когда я читаю, и никто, кажется, не решает их. Особенно в. Вот почему я трачу много времени на то, чтобы исправить корректуру, «заставить ее работать» , поэтому у меня почти не остается времени...
Может быть, вы слишком усложняете? Ищете высокотехнологичные доказательства, когда методы вашего первого года обучения в бакалавриате подходят более точно? Мне трудно сказать с такого расстояния.
+1: «Если вы никогда не заставите себя слишком сильно, вы никогда не узнаете, что такое «слишком сложно».

Этот ответ попытается ответить только на первую часть вашего вопроса. Когда я делала студенческую работу, я сильно набрала вес, так как мой основной способ делать домашнюю работу заключался в том, чтобы просто сидеть и есть чипсы или что-то в этом роде, прижавшись носом к точильному камню. Это, в сочетании с необходимостью постоянно учиться, было очень плохо для моего тела, а также вызывало некоторые проблемы с тревогой в будущем. Примерно на 4-м курсе я начал составлять ежедневные списки дел, которые включали маленькие кусочки вещей, которые не были математическими: я нашел на YouTube короткие (10-15 минут) видеоролики с упражнениями, которые я знал, что у меня есть время, чтобы посвятить их , и я сделал это «100 Push up Challenge», которое вы, вероятно, можете найти в Google (я не совсем туда попал, но мне было очень весело на этом пути!).

По сей день я выделяю время как минимум на 20 минут упражнений каждую ночь (вы удивитесь, насколько вы сосредоточены после этого), и я чувствую себя значительно лучше физически. После того, как вы делаете это в течение месяца или около того, это становится естественным.

Что касается социальной проблемы, то разные люди занимаются разными вещами. Я езжу в школу на велосипеде, поэтому присоединился к группе велосипедистов. Я также нашел ряд других социальных клубов в городе (в то время в Чикаго), которые занимались тем, что мне нравилось. Я был удивлен количеством людей, которые просто искали других людей, с которыми можно было бы поговорить, и не все из них были научными!

Возможно, это не работает для вас, но я хотел поделиться своим опытом на случай, если кто-то найдет его хоть немного полезным.

Спасибо. На самом деле, я набрал вес не из-за такой привычки, а скорее из-за отсутствия физических упражнений, а также из-за того, что еда дает мне энергию, чтобы не спать по ночам, и снижает нервозность. Раньше я был очень спортивным, тренировал ММА и все такое, а теперь все пошло прахом.
Также у меня много интересов и занятий, которыми я занимался и занимался бы сейчас, если бы у меня было больше времени. И у меня была прекрасная светская жизнь до факультета. Я просто захвачен ужасным стрессом и лихорадочным ритмом жизни, от которого я не могу вырваться, не жертвуя своим пониманием и оценками. И вдобавок ко всему, существует реальная вероятность того, что я не смогу сделать карьеру в (академической) математике, так как возможности очень ограничены, а у меня нет связей. Так что все кажется напрасным, что еще больше снижает мою мотивацию.
Я полностью понимаю ваши последние пункты здесь; Многие люди (в том числе один знаменитый!) называли меня посредственным математиком, и это, как правило, меня расстраивало. За последние несколько лет я решил, что даже если для меня нет места в академических кругах, в других местах, безусловно, есть место, и это не значит, что я когда-либо должен перестать заниматься математикой! Думаю, такое мышление поддерживает меня. Я не знаю, что еще вы могли бы сделать, кроме как просто попытаться распланировать свои дни или взять на себя меньшую нагрузку. Однако я не уверен, что любой из них работает для вас.

Вы можете отделить понимание математики от требований ваших классов. Если вам нужно запомнить доказательство для теста, чтобы остаться в школе, сделайте это... но не путайте это с пониманием или с самостоятельной проработкой доказательств. Вы можете обнаружить, что разделение доказательства на части, которые вы понимаете, и части, которые вы не понимаете, упростит процесс работы с доказательством.

Кто знает? Может быть, запомнив доказательство, которое вы не понимаете, вы обнаружите, что можете думать о нем, пока занимаетесь чем-то другим, и, возможно, поймете его в мгновение ока.

Я знаю, что здесь есть один парень, который хочет отговорить вас от занятий математикой... Он абсолютно прав, аспирантура - это очень сложно, аспирантура - это кошмар... Беспокоит то, что он пытается вас подставить. для комфортной жизни... Возможно, это не то, чего вы хотите. Одно я знаю точно: любой человек является «капитаном своей души». Если вы хотите что-то сделать, и если вы не сдаетесь, вы добьетесь своего. Ничто стоящее не дается легко. Вот почему существует такая конкуренция на такие профессии, как математик или повар. Потому что людям, которые хотят этим заниматься, это очень нравится, и для них нет ничего лучше, чем они могли бы сделать. Да, тебе не будут хорошо платить, да какой-нибудь идиот будет ездить на машине лучше тебя. Однако главный вопрос заключается в том, если вы хотите следовать за своей страстью до самого конечного пункта, куда она приведет вас, возможно, в бездну, или если вы хотите никогда не пытаться, и вас будут толкать самые разные люди, которые будут рассказывать, что они думают о жизни и как они думаю, что жизнь должна быть прожита. Я знаю, что ты все равно не слушаешь этих людей, потому что ты подвергаешь сомнению даже математические объекты, ты спрашиваешь «почему?» это все, что вам нужно, в сочетании с практическим подходом к жизни, для карьеры в математике. Я тоже не слушал. Те же люди говорили мне, что я ничего не добьюсь, что математика слишком конкурентоспособна, что мне там не место. Я знаю, что ты все равно не слушаешь этих людей, потому что ты подвергаешь сомнению даже математические объекты, ты спрашиваешь «почему?» это все, что вам нужно, в сочетании с практическим подходом к жизни, для карьеры в математике. Я тоже не слушал. Те же люди говорили мне, что я ничего не добьюсь, что математика слишком конкурентоспособна, что мне там не место. Я знаю, что ты все равно не слушаешь этих людей, потому что ты подвергаешь сомнению даже математические объекты, ты спрашиваешь «почему?» это все, что вам нужно, в сочетании с практическим подходом к жизни, для карьеры в математике. Я тоже не слушал. Те же люди говорили мне, что я ничего не добьюсь, что математика слишком конкурентоспособна, что мне там не место.

На практическом уровне математика не может вызывать увеличение веса; могут быть внешние факторы, на которые следует обратить внимание. Также математика не должна стать причиной потери социальной жизни, возможно, ваши друзья не подходили вам?

Что касается научных кругов, узнайте о практических способах остаться и выжить в научных кругах. Вам понадобится план игры и хорошо продуманная стратегия, которые всегда будут присутствовать в течение следующих 3-5 лет в каждый момент вашей карьеры, вплоть до срока пребывания в должности. Иначе можно остаться без работы. Наиболее важной «валютой» в академических кругах являются публикации, их количество и их качество. Вы должны включить их в свой план очень скоро. Престиж академического учреждения и профессоров, с которыми вы работали, важен на начальном этапе. Посмотрите на успешных математиков (приспособьтесь к тому уровню, на котором вы хотите быть через 10 лет, но всегда старайтесь добиться большего, чем они), их статьи, их карьерный рост и т. д., чтобы получить представление о том, что необходимо. Желаю вам удачи,

Я думаю, что вопрос, не поднятый в приведенных выше различных комментариях, заключается в следующем: какова природа математики и как ее делать? Я считаю, что для любой человеческой деятельности нужно обсуждать методологию , но литературы по этому поводу не так много. Вот статья на эту тему, с которой вы можете согласиться или не согласиться. Вы также можете ознакомиться с предисловиями к моей книге «Топология и группоиды» , которые вы можете просмотреть на сайте amazon.com.

Нужно также исследовать природу собственных талантов по отношению к предмету. Один профессор посоветовал своим студентам делать то, что им кажется самым легким!

Некоторые люди очарованы задачами и комбинаторикой. Меня больше интересуют вопросы, которые я придумал сам, отчасти в надежде, что они удовлетворят трем критериям:

  1. Никто другой не подумал об этом.

  2. Ответ технически не сложен.

  3. Ответ важен.

Все это может казаться маловероятным найти, но если вы не ищете, вы не найдете. Один из моих студентов, Дерек Уоллер, сказал, что ему нравится иметь сотню идей. Если 10% ваших идей хороши, это дает вам десять хороших идей! Пробуйте одну чертову вещь за другой!

Основная тема моих исследований с 1965 года или около того, а именно группоиды, возникла в результате написания книги по топологии. Написание математики для того, чтобы сделать вещи ясными и элегантными, и, таким образом, написание снова и снова, может в конечном итоге заставить вас увидеть, что может быть другой способ делать вещи.

Композитор Равель сказал, что надо копировать. Если у вас есть некоторая оригинальность, это будет видно. Если нет, не беда! На самом деле оригинальная идея может возникнуть только после многократного копирования, так как идея попала в мозг.

Таким образом, мы советовали студентам, что диссертация должна иметь «тезис». Итак, вы должны начать с написания предыстории этой диссертации. При написании вы можете столкнуться с не совсем удовлетворительными битами. Это начало.

Мне посчастливилось наткнуться на эту обширную и гибкую программу теории групп высших измерений , которая заключалась в проверке использования группоидов высших измерений в духе теории групп и, в частности, в связи с теорией гомотопий. Я подозреваю/знаю, что многие «авторитеты» сочли это чепухой, что было недостатком. С другой стороны, это сдерживало конкуренцию.

Я подозреваю, что мой талант — это чувство математической структуры. Мне нравится комментарий Филипа Холла: «Нужно попытаться разработать алгебру, подходящую для геометрии, а не пытаться навязать алгебраическое выражение геометрии определенному способу просто потому, что он доступен».

Я встречал "исследователей", которым либо трудно писать математику, либо не все это делать. Мой совет противоположный: продолжайте писать. Для статьи вы пишете в верхней части листа бумаги: Заголовок. Автор. Введение. Это говорит о том, что вы собираетесь делать. Конечно, у вас могут возникнуть трудности с разделом 1, посвященным основным определениям, но вы всегда можете вернуться к нему позже!

В старой армейской схеме методологии было написано: 1. Общие цели. 2. Текущая ситуация. 3. Ближайшая цель. 4. Метод.

Вы не начинаете с "Метода"!

Я не видел этого ответа, поэтому я решил попробовать.

«Молодой человек, в математике вы ничего не понимаете. Ты просто привыкаешь к ним». Джон фон Нейман

Я всегда стремился понять математику, когда учил ее, но я знаю, когда нужно остановиться и перейти к изучению чего-то другого. Я не думаю, что кто-то может полностью понять математику или физику, если уж на то пошло, у всех великих математиков и физиков до сих пор есть проблемы с пониманием. Кажется, что уровень понимания, который вы ищете, слишком метафизичен. Некоторые из лучших математиков обладали потрясающей памятью, это, несомненно, помогало им, потому что они могли использовать ее для развития дальнейшего анализа и т. д.

«Кроме того, у меня проблема со слабой памятью. Я многое забываю».

Вы пробовали использовать программу для карточек? Есть программа , где можно добавлять вопросы. Если вы правильно ответили на вопрос, программа задаст вам вопрос повторно через 2 дня. Если вы ответите правильно через 2 дня, он снова спросит вас через 4 дня. После этого интервал вырастет до 8, 16 и так далее. Я нашел это полезным. Вы также можете установить собственный интервал. Программа принимает математические символы.

Кроме того, не добавляйте в программу ВСЕ, что вы хотите запомнить. Вы все равно многое вспомните. Хитрость заключается в том, чтобы выяснить, какие из них вам нужно добавить.


Говоря о том, как избавиться от соблазна постоянно заниматься математикой, я хотел бы упомянуть концепцию предельной полезности. Предельная полезность чего-либо (скажем, унции воды) — это то, насколько полезно приобретение одной дополнительной единицы. Если вы обезвожены, предельная полезность унции воды очень высока. По мере того, как вы пьете все больше и больше воды, ее предельная полезность уменьшается. То же самое происходит и с математическими знаниями. Даже если вы потребляете новую математику в течение многих лет и стремитесь получить больше, наступит момент, когда ее предельная полезность уменьшится, и вы сможете легко сосредоточиться на других вещах. Вы просто еще не знаете, что это за точка, и, может быть, вы не достигнете ее даже через годы, но, исходя из моего опыта, я ожидаю, что вы ее когда-нибудь достигнете.

На самом деле чрезмерная гидратация может быть фатальной, если вы выпьете слишком много, вы навредите своему телу, возможно, вплоть до смерти. Чтобы довести точку предельной полезности до сознания, если вы слишком много занимаетесь математикой, вы можете в конечном итоге вообще ничему не научиться.

Кое-что может быть полезно, когда солдаты спецназа тренируются, под названием Devil Training.

Кажется, веские причины связаны с преодолением предела возможностей для быстрого расширения зоны комфорта и оснащением чрезвычайно сложными практиками в очень сложных условиях, а НЕ в простых.

В некоторых частях, таких же, как и в хорошей математической подготовке, можно было быстро усвоить как широкие, так и важные знания и быстро двигаться дальше, интенсивно практиковаться в продвинутой и сложной «среде», чтобы одновременно тренировать как элементарные, так и продвинутые навыки. Никогда не оставайтесь на легком уровне, а просто идите дальше, даже если какие-то навыки/знания плохо усваиваются, так как по мере продвижения дальше будет лучше. Есть только тонкая грань между Мудростью и Глупостью, мудрые люди впитывают и быстро продвигаются вперед, играя со знанием, как плавающая рыба, и продолжают углублять понимание; в то время как глупцы только пассивно страдают от боли и повторяют все механически, не думая.

В то время как некоторые части, по-видимому, противоположны, в военной подготовке это подчиняться приказам, не задавая вопросов, но в математике (естествознании) это нарушать приказы, бесконечно задавать вопросы. Плюс немного страсти, любопытства, самопроизвольной готовности исследовать и немного удачи, это может создать очень красивую дорогу в математическом мире.

Как CPA, который сейчас возвращается в школу, чтобы получить степень магистра компьютерных наук (и, следовательно, несколько математических курсов по пути), я могу косвенно относиться к вашей проблеме; тем не менее, я верю, что моя точка зрения актуальна и вполне может быть вам полезна.

Я был таким же, как вы, когда учился в бакалавриате по бухгалтерскому учету. Я буквально каждую минуту своего бодрствования в течение последних двух лет учебы — семь дней в неделю — готовился к занятиям. Как и вы, мной движет «почему»; не только "как". В сфере бизнеса справедливо и то, что большинство студентов не задаются вопросом, почему решение верно; они просто признают, что это правда, и запоминают шаги для выработки соответствующего решения.

В отличие от вас, я не считаю себя лишь незначительно талантливым или в лучшем случае посредственным; и, на самом деле, я призываю вас серьезно подумать над комментарием более раннего автора, который бросил вызов вашему самоуничижительному мнению, сказав, что вы действительно можете быть талантливым и, следовательно, не делать преждевременного суждения о себе. Пословица: Как я могу построить дом, если я не верю, что я тоже могу? Не будет пустой тратой времени подумать о том, как ваше внутреннее убеждение о себе может значительно повлиять на а) вашу успеваемость и б) уровень вашей личной тревожности. Пожалуйста, примите это близко к сердцу.

Сделав сейчас такие заявления, я также хотел бы сказать, что с практической точки зрения я полностью «попадаю» туда, где вы сейчас находитесь. Вам нужна перспектива завтрашнего дня, сегодня, и тем самым убедиться, что сегодня вы не сделаете глупый выбор, который будет стоить вам завтра. Мой совет — следовать глубочайшей страсти своего сердца сегодня и довериться «завтра», чтобы проработать другие детали.

Например, если занятие математикой действительно является высшей страстью вашей жизни, тогда посвятите себя этому пути, независимо от того, станете ли вы штатным профессором или вместо этого окажетесь в какой-то области прикладной математики... или даже в разработке программного обеспечения. Каждый должен делать все возможное и стремиться к наивысшему достижению. Вы всегда будете счастливы, стремясь сделать все возможное, но в то же время следуя глубочайшей страсти сердца.

Конечно, в жизни есть нечто большее, чем «X» (например, математика в вашем случае), поэтому мы должны также развивать и другие части нашего существа. Даже наши низменные желания учат нас этому, настаивая каждые двадцать четыре часа на кормлении, отдыхе, уборке и облегчении. А как насчет наших, высших, эмоциональных и духовных потребностей? Разве это просто тщеславие — искать человечности (общения)? Как и в случае предположений, присущих математике, наши плотские и психологические желания также обеспечивают самоочевидные предположения, помогающие в построении процесса нашего существования. Прислушивайтесь к своему и действуйте соответственно, но помните, что не каждый дом имеет одинаковую форму, размер и симметрию; поэтому вам может потребоваться очень мало или очень много таких переменных «у» при решении фактора «х» вашей жизни.

Установите свои приоритеты. Получите календарь. Поместите в него упражнения на полчаса 3 раза в неделю. Это лучше, чем ничего. Не делайте ничего, что вас полностью утомит. Запланируйте время, чтобы пообщаться с людьми. Запланируйте другие приоритеты, такие как сон. Затем, сколько бы времени ни осталось, а это, вероятно, было бы довольно мало, займитесь математикой. Если вы все еще чувствуете то же самое, запланируйте больше упражнений/сна/прогулок или чего-то еще.

Многие забывают математику. Я хотел изучать алгебру. У меня был реальный анализ 1 и 2 (уровень выпускника) один год, и я потратил на них столько времени, что у меня не было времени на алгебру. Я не мог вспомнить алгебру, которая была бы действительно важна. Настоящий анализ мне даже не важен. Я должен был лучше расставить приоритеты и тратить меньше времени на реальный анализ и больше времени на алгебраические вещи.

В том же году я провел так много времени вдали от своей семьи, что было ужасно. Я сейчас в основном работаю 8-5 и иду домой. И, может быть, в субботу я работаю несколько часов. Иногда я пытаюсь получить больше работы. Но моя семья важнее этой математики, поэтому меня не волнует, потребуется ли больше времени, чтобы получить высшее образование, или я не так хорошо учусь.

Я не пытаюсь быть грубым или что-то в этом роде, но часто не помогает простое произнесение императива вроде «запланируйте время для упражнений». Обычно люди спрашивают , как планировать такое время.
@EMS Не хочу быть грубым, но, глядя на контекст, вы можете видеть, что то, что я сказал, имело смысл. Он говорит, что все свое время посвящает математике. Я говорил ему запланировать время, когда он не занимается математикой, и заниматься математикой в ​​остальное время. Затем он делает то, чего раньше не делал.
Верно, но это не помогает объяснить, как тратить время на другие вещи. Плакат даже не задал бы вопрос, если бы он или она уже не знали, что им следует уделить время этим другим вещам. Но это сильно отличается от фактического курса действий, который дает детали реализации о том, как заставить себя на самом деле посвятить себя этому делу. Более уместным советом было бы изучить состояние искусства самопомощи , такие вещи, как уравнение прокрастинации или предварительное обязательство.
@EMS Если у вас есть такой отличный ответ, ответьте на вопрос сами.
Я сделал это еще до того, как прокомментировал ваш ответ ( ссылка ). Извините, если мои комментарии вас обидели. Это была просто конструктивная, вежливая критика.

Вы настолько хороши, насколько хорошо ваше самое слабое звено. Многие студенты и люди, занимающиеся математикой, недооценивают важность хорошей формы. Ваша продолжительность концентрации увеличивается, и ваш сон часто становится лучше, когда вы тренируетесь и питаетесь здоровой пищей. Я получил так много озарений после того, как пробежал несколько миль, отдышавшись после того, как застрял на чем-то.

Что касается общественной жизни; семья важна, близкие друзья тоже. Я никогда не посещал какие-либо «социальные» мероприятия в университете, это просто разрушительно, нет ничего хорошего в том, чтобы напиться и не спать всю ночь. Может быть, пообедать с несколькими студентами, выпить пива и просто расслабиться.

Что касается набора веса: прочтите несколько работ Артура Де Вани и ознакомьтесь с его диетой. Люди едят постоянно и ожидают, что не наберут вес. Де Вани предполагает, что в некоторые дни после здорового, плотного завтрака можно ничего не есть и ложиться спать голодным. Часть «засыпать голодным» вызывает большие возражения со стороны людей, но наши тела еще не эволюционировали из своей структуры охотников-собирателей. Когда люди охотились, они могли днями обходиться без еды. Наш метаболизм направлен на то, чтобы после хорошего улова объедаться (переедать) и ходить голодным до этого улова. Постоянно есть - не наш путь. Попробуйте это, и вы станете счастливее, ваше тело, когда оно голодно, даже почувствует облегчение, потому что ему нужно будет работать на одну вещь меньше. В дни, когда вы не едите (после завтрака),

Возможно, один из лучших советов, которые я слышал относительно этого типа дилеммы (не только в математике, но и во всех практиках), заключается в том, что если это не весело, вам нужно изменить это.

Похоже, ваши математические интересы стали скорее одержимостью, чем развлечением. Математика явно соответствует тому, как вы думаете, и вы стремитесь получить удовлетворение от ее понимания, но вы зашли слишком далеко.

Два момента:

  1. Вы не все поймете
  2. Это нормально не все понимать

Математика — это инструмент и редко стиль жизни. Возможно, вам покажется странным всю свою жизнь посвятить пониманию и разработке идеального набора гаечных ключей. Поступая таким образом, вы можете придумать лучший набор, но вы упускаете все ремонтные работы, сборку и другие продуктивные действия, которые вы можете сделать с набором, который у вас есть.

Используйте свою математику и позвольте пониманию, которое вы получаете от ее использования, способствовать дальнейшему пониманию тех практик, которые вам интересны. Если это весело, сделайте это, а если не весело, измените ситуацию. Сделайте все это математическим по пути, только не застревайте в математике.

Агония по поводу «Как изучать математику?» не так реален, как представляется. Прежде чем приступить к изучению темы, следует уделить десять процентов времени разработке стратегии завершения главы. Никогда ничего не принимайте на веру в математике. Не полагайтесь на доказательство теоремы, данное в книгах, со слишком большим количеством «ясно». Не полагайтесь на авторитет профессора. Они редко идеальны для подражания. Просто попытайтесь найти собственное доказательство, поместив перед глазами каждое определение и предыдущие результаты, написанные на широком листе бумаги. Никогда не полагайтесь на память в отношении определения или формулировки сложной теоремы при использовании чего-либо. В основном они обманывают новичка. Просто смотрите на это без стыда. Таким образом, вы получите удовольствие и сформируете привычку доказывать что-либо самостоятельно. Только в крайнем случае прочтите некоторые части доказательства. Не сходите с ума для решения Проблем. Они редко вызывают затруднения у студента, прошедшего процедуру, описанную мной при заполнении главы. Сохраните «обнаруженные вами доказательства» для будущих нужд. Вы будете поражены, заметив, что после обучения таким образом вы становитесь смелее с каждым днем. Но даже в этом случае, если вы попадете в беду, вам следует уделять больше времени тому, чтобы сделать ваши предпосылки живыми. Откажитесь от идеи отказаться от математики. Только удачливые люди посвящают свою жизнь математике. Молодые люди не так нуждаются в физических упражнениях, как пожилые. Ваш приоритет — математика, а не что-то еще. Если вы выйдете из-за стола, не завершив работу, вам придется потратить вдвое больше времени на перепросмотр в следующий раз, когда вы сядете. Таким образом, вы будете большим неудачником в долгосрочной перспективе. Так что покидайте учебный стол только после выполнения поставленной задачи. Они редко вызывают затруднения у студента, прошедшего процедуру, описанную мной при заполнении главы. Сохраните «обнаруженные вами доказательства» для будущих нужд. Вы будете поражены, заметив, что после обучения таким образом вы становитесь смелее с каждым днем. Но даже в этом случае, если вы попадете в беду, вам следует уделять больше времени тому, чтобы сделать ваши предпосылки живыми. Откажитесь от идеи отказаться от математики. Только удачливые люди посвящают свою жизнь математике. Молодые люди не так нуждаются в физических упражнениях, как пожилые. Ваш приоритет — математика, а не что-то еще. Если вы выйдете из-за стола, не завершив работу, вам придется потратить вдвое больше времени на перепросмотр в следующий раз, когда вы сядете. Таким образом, вы будете большим неудачником в долгосрочной перспективе. Так что покидайте учебный стол только после выполнения поставленной задачи. Они редко вызывают затруднения у студента, прошедшего процедуру, описанную мной при заполнении главы. Сохраните «обнаруженные вами доказательства» для будущих нужд. Вы будете поражены, заметив, что после обучения таким образом вы становитесь смелее с каждым днем. Но даже в этом случае, если вы попадете в беду, вам следует уделять больше времени тому, чтобы сделать ваши предпосылки живыми. Откажитесь от идеи отказаться от математики. Только удачливые люди посвящают свою жизнь математике. Молодые люди не так нуждаются в физических упражнениях, как пожилые. Ваш приоритет — математика, а не что-то еще. Если вы выйдете из-за стола, не завершив работу, вам придется потратить вдвое больше времени на перепросмотр в следующий раз, когда вы сядете. Таким образом, вы будете большим неудачником в долгосрочной перспективе. Так что покидайте учебный стол только после выполнения поставленной задачи.